Download as pdf or txt
Download as pdf or txt
You are on page 1of 42

‫اﻟﻠﻬﻢ اﺳﺘﻮدﻋﻚ ﻣﺎ ﻗﺮأت وﻣﺎ ﺣﻔﻈﺖ‬

‫اﺳﺌﻠﺔ وزارة اﻟﺼﺤﺔ ﻟﻼﻟﺘﺤﺎق‬ ‫وﺗﻌﻠﻤـﺖ ‪ ،‬ﻓـﺮده ﻟـﻲ ﻋﻨـﺪ ﺣﺎﺟﺘـﻲ اﻟﻴﻪ ‪,‬‬

‫ﺑﺒﺮﻧﺎﻣﺞ اﻻﻗﺎﻣﺔ واﻻﻣﺘﻴﺎز‬ ‫إﻧـــﻚ ﻋﻠﻰ ﻣﺎ ﺗﺸﺎء ﻗﺪﻳﺮ‬

‫) ‪( 2018 -2013‬‬
‫اﺧﻮﻛﻢ اﻟﺪﻛﺘﻮر ‪ :‬اﺳﺎﻣـﻪ اﻟﺨﺰاﻋﻠـﻪ‬
‫ﻗﻤﺖ ﺑﺎﻋﺎدة ﺟﻤﻊ وﺗﻨﺴﻴﻖ‬ ‫ﺑﻔﻀﻞ ا‬
‫‪0799430239‬‬
‫اﺳﺌﻠﺔ اﻟﺪورات اﻟﺴﺎﺑﻘﺔ‪ ,‬اﻟﺼﺎدرة ﻣﻦ‬ ‫ﻃـــﺐ ﻋـــﺎم وﺟﺮاﺣــﻪ‬
‫وزارة اﻟﺼﺤﺔ ﻻﻣﺘﺤﺎﻧـــﺎت اﻻﻗﺎﻣـــﺔ‬

‫) ‪-2013‬‬ ‫واﻻﻣﺘﻴــﺎز ﻟﻼﻋـــﻮام‬

‫‪ , ( 2018‬ﺑﻌﺪ ﺟﻬـــﺪ وﻋﻨــﺎء ‪ ,‬وﺣﻠﻬﺎ‬

‫ﺑﺎﻻﺳﺘﻌﺎﻧﺔ ﺑﺎﻟﻤﺮاﺟﻊ واﻟﻜﺘﺐ اﻟﻤﺨﺘﻠﻔﺔ‬

‫ﻟﺘﻮﺧﻲ اﻟﺪﻗﺔ ﻓﻲ ﻧﺸﺮ اﻟﻤﻌﻠﻮﻣﺔ‬

‫اﻟﺼﺤﻴﺤــﺔ ﻟﻠﺰﻣﻼء واﻟﺰﻣﻴﻼت ‪ ,‬ﻓﺈن )‬

‫‪ ,‬وان اﺳﺄت ﻓﻤﻦ‬ ‫اﺣﺴﻨﺖ ﻓﻤﻦ ا‬

‫ﻧﻔﺴﻲ ( ‪ ,‬واﻧﻲ اﻗـــﺪم ﻫﺬا اﻟﻌﻤﻞ ﺧﺎﻟﺼﺎ‬

‫ﺗﻌﺎﻟﻰ ‪ ,‬راﺟﻴﺎ اﻟﻤﻮﻟﻰ اﻟﻘﺒﻮل ‪,‬‬ ‫ﻟﻮﺟﻪ ا‬ ‫*****‬


‫وان ﻳﻨﺎل اﻋﺠﺎﺑﻜﻢ ‪.‬‬

‫‪OBS & GYN‬‬


* All of the following are absolute indications for
cesarean section except ?

*****
A . Cord prolapse
B . No progression of labor
C . Previous one cesarean section
D . Cervical obstruction
E . Transverse lie

* Fetal tachycardia on CTG may be caused by all


of the following except ?
A . Maternal fever
B . Prematurity

‫ﻧﺴﺨﺔ ﻣﺪﻗﻘﺔ‬ C . Thyrotoxicosis


D . Maternal SLE
E . Maternal chorioamnionitis

* In the reproductive period the PH of the vagina


is ?
A.6_8
B.5_6
C.4_5
D.3_4
* The smallest diameter of the true pelvis is ? E.2_3
A . True conjugate * A 26 years old lady , had forceps delivery , after
B . Obstetric conjugate uneventful delivery of the placenta , she started
C . Diagonal conjugate to have heavy vaginal bleeding , the uterus was
D . Transverse diameter firm , the most likely cause for the bleeding is ?
E . Oblique diameter A . Uterine atony
B . Retained placenta
* Regarding gestational trophoblastic neoplasia C . Infective uterus
( GTN) all of the following are correct except ? D . Genital trauma
A . Complete mole associated with presence of E . Coagulopathy
fetal and embryonic tissue .
B . Incomplete mole associated with focal * The most common presenting symptom of
swelling of chorionic villi . PID
C . Karyotype of partial mole usually triploidy . A . Vaginal discharge
D . Theca lutein ovarian cysts develop in about B . Lower abdominal pain
half of patients with complete mole . C . Fever
E . Snow storm appearance on ultrasound is D . Dyspareunia
characteristic for complete mole . E . Menorrhagia

* A healthy woman complaining of crying , * If a pregnant lady had last menstrual period 1
insomnia , feeling low , mood lability 3 day after October , presented at 1 July for her routine
normal delivery , the most likely cause is ? antenatal care , and her exam showed symphysis
A . Postpartum depression fundal height of 31 cm , all of the following could
B . Postpartum psychosis be possible causes except ?
C . Postpartum psychosis A . Wrong date of last menstruation .
D . Postpartum neurosis B . Rupture of membrane .
E . Postpartum schizophrenia C . Fetal demise .
D . Transverse lie .
* Women with uterine fibroid can present with all E . Hydatidiform mole .
of the following except ?
A . Menorrhagia * Regarding ectopic pregnancy one of the
B . Increased frequency following is true ?
C . Infertility A . A positive pregnancy test is enough for
D . Abdominal and pelvic pain diagnosis .
E . Spasmotic dysmenorrhoea B . Vaginal bleeding is always present .
‫ اﺳﺎﻣﻪ اﻟﺨﺰاﻋﻠﻪ‬: ‫اﻟﺪﻛﺘﻮر‬ ‫اﺳﺌﻠﺔ اﻟﻨﺴﺎﺋﻴﺔ واﻟﺘﻮﻟﻴﺪ‬
Page 2
C . Surgical treatment in most of the case's . C . Thyroid function test
D . Most cases developed rupture of tubes . D . Progestin challenge test
E . IUD is risk factor . E . B_ HCG serum

* You are examining a 34 year old woman * In patients with hypogonadotropic


gravida 3 , Para * at 38 weeks gestation . She is hypogonadism , the typical pattern of FSH _ LH
in labor 5 cm cervical dilation . There is no fetal is ?
part in the pelvis . Ultrasound reports notes a A . Decreased LH , increased FSH .
transverse lie with fetal back up towards the B . Decreased FSH , increased LH .
maternal arms . The best management is ? C . Decreased both LH and FSH .
A.Expectant management anticipating D . Increased both LH and FSH .
spontaneous vagina delivery . E . Normal LH , decreased FSH .
B . Tocolysis .
C . External version . * An arterial anastomosis is formed in the broad
D . Cesarean section . ligament by branches of the ?
E . Expectant management expecting forceps A . Tubal artery and ovarian artery .
rotation after complete dilation . B . Uterine artery and tubal artery .
C . Uterine artery and ovarian artery .
D . Tubal artery and uterine artery .
E . Hemorroidal artery and uterine artery .

* A 35 year old complains of increasing


dysmenorrhoea and pelvic pain . She has not * Regarding breastfeeding , one of the following
become pregnant despite 3 years of unprotected is false ?
intercourse . Her pelvic exam demonstrates A . Can inhibit ovulation .
tenderness and nodularity over her uterosacral B . Is not recommended in case of HIV infection .
ligament and 4 cm right ovarian cyst . Most likely C . Is not absolutely contraindicated in case of
diagnosis is ? HBV infection .
A . Adenomyosis D . Methimazole is preferable in case of
B . Endometriosis hyperthyroidism .
C . Chronic ectopic pregnancy E . Is contraindicated in case of Galactosemia .
D . Chronic PID
E . Chronic pelvic congestion syndrome * A 28 year old patient complains of Amenorrhea
after D and C . She denies any other complaints
* Regarding bacterial vaginosis one is true ? and did not require a blood transfusion at the
A . It is caused by Trichomonas vaginalis . time and after delivery . Which of the following is
B . 50% of case's are silent and no treatment most likely diagnosis ?
required . A . Gonadal dysgenesis
C . Yellow frothy vaginal discharge . B . Sheehan syndrome
D . Petechiae on vagina and cervix . C . Kallmann syndrome
E . Intense genital pruritus . D . Mayer Rokitansky Kuster Hauser syndrome
E . Asherman syndrome
* All of the following are recommended
treatment option for patient with endometriosis * A 47 year old female who completed her
except ? family , presented to the clinic with new onset of
A . NSAIDs heavy menstrual bleeding , endometrial biopsy
B . OCP showed no malignancy . All are true in
C . GnRH agonist management except ?
D . Estrogen therapy A . Tranexamic acid can help in control of
E . Mirena IUD symptoms .
B . Mirena is one of the options .
* In Amenorrhea , all of the following may be C . Combined OCP is useful in management .
relevant except ? D . NSAIDs have a great benefits .
A . BMI E . Hysterectomy the best option .
B . Skull X_Ray
‫ اﺳﺎﻣﻪ اﻟﺨﺰاﻋﻠﻪ‬: ‫اﻟﺪﻛﺘﻮر‬ ‫اﺳﺌﻠﺔ اﻟﻨﺴﺎﺋﻴﺔ واﻟﺘﻮﻟﻴﺪ‬
Page 3
* Before puberty the PH of the vagina is ? E . Rapid and prolonged labor
A . 6.8 _ 7.2
B . 5.8 _ 6.2 * The age of menopause is ?
C . 4.5 _ 5.2 A . Related to regularity of menstrual cycle .
D . 3.5 _ 4.2 B . Related to last time of pregnancy .
E . 7.4 _ 7.6 C . Early menopause in case of smoking female .
D . Delayed menopause in case of alcoholic
* One of the following is not a recognized cause female .
of delayed Onest of puberty ? E . Related to race and genetic factor .
A . Turner syndrome
B . Mccune Albright syndrome * Menarche usually occurs between which years
C . Anorexia nervosa of age ?
D . Kallmann syndrome A . 8 _ 10 years
E . Craniopharyngioma B . 11 _ 13 years
C . 14 _ 16 years
* A 19 weeks pregnant woman with Hb 9 g/dL , D . 16 _ 18 years
the cause was iron deficiency , the next step is ? E . 18 _ 20 years
A . Do nothing because it is physiological .
B . Give IV iron . * During pregnancy , what is the safest analgesic
C . Start IM iron . with the least fetal effect ?
D . Give high dose iron orally . A . Aspirin
E . Give blood . B . Acetaminophen
C . Ibuprofen
D . Gabapentin
E . Diclofenac sodium
* One of the following is not a normal 3* A 75 year old woman has bilateral , solid
physiological changes in pregnancy ? adnexal masses . Mammography is normal , GI
A . Increase in the heart rate . studies show a stomach lesion . Which of the
B . Increase in white cell count . following is the most likely diagnosis ?
C . Increase in plasma volume . A . Pick adenoma
D . Increase in glomerular filtration . B . Krukenberg tumor
E . Increase in gastric emptying C . Brenner tumor
D . Struma ovarii
* Regarding infertility , one is false ? E . Carcinoid tumor
A . Infertility is inability of couples to conceive
after one year of regular unprotected sexual * The relationship of the long axis of the fetus to
intercourse . the long axis of the mother is called ?
B . Primary infertility it is infertility in the context A . Lie
of no prior pregnancy at all . B . Position
C . Female factor is a major factor of infertility . C . Presentation
D . Common cause of female factor is PCOS . D . Attitude
E . Clomiphene citrate is used in case of tubal E . Axis
cause of infertility
* You are checking a term patient in labor . The
* All of the following are sexually transmitted examination of the fetal presentation feels
disease except ? unusual . Which of the following would be
A . Toxoplasmosis incompatible with spontaneous delivery ?
B . Trichomoniasis A . Occiput anterior
C . HPV B . Occiput posterior
D . HSV2 C . Mentum posterior
E . Chancroid D . Occiput transverse
E . Mentum anterior
* All of the following are causes of atonic
postpartum hemorrhage except ? * When performing clinical pelvimetry in a
A . Multiple gestation gynecoid pelvis . The diagonal conjugate should
B . Multiparity be at least how many centimeters ?
C . Fibroid with pregnancy A . 7.5
D . Preeclampsia B . 9.5
‫ اﺳﺎﻣﻪ اﻟﺨﺰاﻋﻠﻪ‬: ‫اﻟﺪﻛﺘﻮر‬ ‫اﺳﺌﻠﺔ اﻟﻨﺴﺎﺋﻴﺔ واﻟﺘﻮﻟﻴﺪ‬
Page 4
C . 11.5 B . Endodermal sinus tumor
D . 13.5 C . Embryonal carcinoma
E . 15.5 D . Choriocarcinoma
E . Mature teratoma
* During normal pregnancy a lowered
hemoglobin is a physiological finding . What is * A 40 year old woman is found on pelvic exam
the major cause ? to have an enlarged uterus . Ultrasound reveals a
A . Low iron store's . well circumscribed intramural mass consistent
B . Blood lost to the placenta and fetus . with the leiomyoma . The patient ask what is the
C . Increased plasma volume . incidence of sacromatous degeneration in a
D . Increased red cell destruction . uterine leiomyoma ?
E . Decreased red cell production from bone A . < 1%
marrow . B . < 3%
C . 10 %
* Regarding immunization during pregnancy , D . 15 %
which of the following vaccines would be the E . 30 %
safest to receive during pregnancy ?
A . Mumps * A 38 year old multigravid complains of the
B . Polio painless loss of urine , beginning immediate with
C . Rabies coughing , laughing , lifting , or straining . Which
D . Rubella of the following is this history most suggestive of
E . Rubeola A . Fistula
B . Stress incontinence
C . Urge incontinence
D . UTI
* An asymptomatic 25 year old pregnant woman E . Overflow incontinence
is found on routine prenatal screening at 14 4* A 21 year old woman has Amenorrhea , mild
weeks gestation bacteriuria ( 100000 colonies / vaginal spotting , pelvic pain , and left shoulder
ml ) . What is her risk of developing pain . Her vital signs are BP 90/40 mmhg , HR
pyelonephritis if untreated ? 120 bmp , RR 23 bmp . Abdominal exam shows
A . 5 _ 10% LLQ tenderness with rebound . Pelvic exam
B . 20 _ 30 % shows painful 4 cm left adnexal mass . A serum
C . 40 _ 50 % pregnancy test is positive . A hemotocrit is 21% ,
D . 60 _ 70 % which of the following the best next step ?
E . 90 _ 100 % A . Observation
B . Estrogen therapy
* Which of the following types of vulvar cancer C . Progesterone therapy
occurs most commonly ? D . Methotrexate therapy
A . Pagets E . Immediate laparotomy
B . Squamous
C . Melanoma * The definitive treatment of preeclampsia is ?
D . Adenocarcinoma A . Bed rest
E . Basal cell B . Pregnancy termination
C . Antihypertensive drugs
* Which of the following factors is protective D . Low salt diet
against endometrial hyperplasia ? E . Magnesium sulfate
A . Obesity
B . Tamoxifen * A patient with Sheehan syndrome wishes to
C . OCP have another child . Ovulation can be induced
D . Early Menarche and late menopause using which of the following hormonal therapy ?
E . Unopposed estrogen therapy A . Low dose estrogen therapy .
B . Human menopausal gonadotropins .
* A 18 year old female presents with abdominal C . Pulsatile GnRH .
pain . During the emergency department work up D . Clomiphene citrate .
an adnexal mass is found . You suspect germ E . High dose estrogen therapy .
cell tumor . Her preoperative discussion that the
most common germ cell tumor is ? * A 22 year old woman with a history of ectopic
A . Dysgerminoma pregnancy presents for contraceptive
‫ اﺳﺎﻣﻪ اﻟﺨﺰاﻋﻠﻪ‬: ‫اﻟﺪﻛﺘﻮر‬ ‫اﺳﺌﻠﺔ اﻟﻨﺴﺎﺋﻴﺔ واﻟﺘﻮﻟﻴﺪ‬
Page 5
counseling . Which of the following contraceptive C . Increased level of FSH : LH > 3:1 .
methods would be relatively or absolutely D . Can be treated by laparoscopic drilling
contraindicated E . No role of metformin in treatment .
A . OCP
B . Mini pills * 25 years old female patient pregnant 37
C . Tubal ligation weeks , she completed her pregnancy and
D . IUCD delivery uneventfully , after 3 days of delivery, she
E . Male condoms is complain of persistent vaginal bleeding , was
done ultrasound _ their is no placental remnants
* A 20 year old patient complains of painful in the uterus , laboratory results was B_HCG is
vulvar ulcers present for 72 hours . Examination positive and very high bleeding is continuous ,
reveals three tender , punched out lesions with a the most likely diagnosis is ?
yellow exudate but no induration . Which of the A .Hydatiform mole
following most likely diagnosis ? B .Ectopic pregnancy
A . Chancroid C .Choriocarcinoma
B . Granuloma inguinale D .Cervical trauma during delivery
C . HSV2 E .Cervical cancer
D . Lymphogranuloma venereum
E . Syphilis * Most common cause of first trimester abortion
is ?
* The three component of the cervical exam in A .Chromosomal abnormalities
labor include ? B .Septate uterus
A . Dilation , presentation , effacement . C .STORH infection
B . Effacement , station , position . D .Cervical incompetence
C . Dilation , effacement , station . E .DM
D . Dilation , descent , presentation .
E . Presentation , descent , effacement
* In a young obese , chronically anovulatory
woman with an elevated LH:FSH ratio and * One is true about Puerperium ?
polycystic appearing ovaries , which of the A . This period last for 50 days postpartum .
following is the preferred initial method of B . Subinvolution of the uterus indicate
ovulation induction ? endometritis
A . Metformin C . First 1_3 days postpartum vaginal secretions
B . Human menopausal gonadotropins is Lochia alba .
C . Pulsatile GnRH D . Hydronephrosis disappear by 3_6 weeks
D . Clomiphene citrate postpartum .
E . Bromocriptine E . Most common cause of postpartum
hemorrhage is genital tract trauma .
* All are wrong about menstrual cycle except ?
A . Amenorrhea is a painful menstruation . * 28 years old female pregnant smoker , 35 week
B . Menorrhagia is a irregular periods that last gestational age , suddenly she is complain of
less than 7 days . painful vaginal bleeding , P.V examination no
C . Duration of menstrual cycle vary from 21 days cervical dilation , on CTG their is fetal
to 35 days Bradycardia . The most likely diagnosis is ?
D . Day 14 is always day of ovulation . Placenta previa
E . Their is four endometrial phases during Abruption placenta
menses . Vasa previa
Placenta accreta
* Most common site of ectopic pregnancy is ? Uterine rupture
Uterus
Ovarian * All are causes of IDA in mother except ?
Peritoneal Abruption placenta
Cervical Placenta previa
Ampulla of fallopian tube Uterine rupture
Vasa previa
* One is true about PCOS ? Placenta accreta
A . Most common cause of primary Amenorrhea .
B . More risk for ovarian cancer .
‫ اﺳﺎﻣﻪ اﻟﺨﺰاﻋﻠﻪ‬: ‫اﻟﺪﻛﺘﻮر‬ ‫اﺳﺌﻠﺔ اﻟﻨﺴﺎﺋﻴﺔ واﻟﺘﻮﻟﻴﺪ‬
Page 6
* One of the following is a cause of E . High parity
Polyhydramnios ?
A .Congenital anomalies * Trichomonas vaginalis is ?
B .Potter syndrome A . Viral infection .
C .Polycystic kidney disease B . Zoonotic disease .
D .Hypoplasia of the lung C . Fungal infection .
E.Indomethacine intake during pregnancy D . STD
E . Premalignant infection .
* Cervical canal is lined with ?
A . Stratified squamous epithelium . * The commonest infection during pregnancy is
B . Ciliated columnar epithelium . A . GIT infection .
C . Non_ciliated columnar epithelium B . Upper respiratory tract infection .
D . Transitional epithelium . C . Lower respiratory tract infection .
E . Simple squamous epithelium . D . Genital tract infection .
E . UTI
* Most common physiological microorganism
lives in the vagina of pregnant female is ? * All are chromosomal disease except ?
A . Lactobacillus acidophilus A . Turner syndrome .
B . Streptococcus Hemolytic type A . B . Celiac disease
C . Streptococcus Hemolytic type B . C . Down syndrome .
D . Candida albicans . D . Cystic fibrosis .
E . Chlamydia Trachomatis . E . Polycystic kidney disease .

* Normal physiological vagina is lined with ? * All are true matching except ?
A . Simple columnar epithelium . A . Patau syndrome _ XY , 47 , +13 .
B . Simple squamous epithelium . B . Fragile x syndrome _ XY , 46
C . Transitional epithelium . C . Klinefelter syndrome _ XXY , 47 .
D . Stratified squamous epithelium D . Turner syndrome _ X0 , 45 .
E . Glandular columnar epithelium . E . Down syndrome _ XY , 47 , +21 .
* The normal physiological base line of heart * All are true about external Cephalic version
rate of the fetus is ? except ?
A . 110 _150 A . Should be done at term pregnancy , equal or
B . 100 _ 140 . more than 37 weeks gestational age .
C . 90 _130 . B .It is not an indication for anti_D administration
D . 120_170 . to the RH_negative mother
E . 100_130 . C . Recommended to do in case of breech baby .
D . It is risk for Abruption placenta and preterm
* Shoulder dystocia is mostly seen in newborn labor .
due to ? E . Should be done by experience obstetrician .
A . Congenital hypothyroidism .
B . Digeorge syndrome . * Anti_D should be given in all of the following
C . Down syndrome . cases except ?
D . Multiple gestation . A . Abruption placenta
E . Infant of diabetic mother B . Threatened abortion
C . External cephalic version
* Site of effusion of two pubis bone is called ? D . Gestational DM
A . Pubic crest . E . Rh negative mother and Rh positive husband ,
B . Pubic Symphysis . after normal vaginal delivery .
C . Coccyx .
D . Ischial spine . * One is not an indication for induction of labour
E . Pelvic brim . A . Post_term pregnancy .
B . Abortion incomplete .
* All are risk factors for endometrial cancer C . Fetal death in utero .
except ? D . Complete placenta previa
A . Obesity . E . Premature rupture of membrane .
B . Late menopause .
C . Endometrial hyperplasia . * 25 years old female pregnant , 12 weeks
D . Tamoxifen usage . gestational age , presented to gynecologist to
‫ اﺳﺎﻣﻪ اﻟﺨﺰاﻋﻠﻪ‬: ‫اﻟﺪﻛﺘﻮر‬ ‫اﺳﺌﻠﺔ اﻟﻨﺴﺎﺋﻴﺔ واﻟﺘﻮﻟﻴﺪ‬
Page 7
follow up her pregnancy , at this first visit should C . Head circumference
be done all of the following except ? D . Occipitofrontal diameter
A . Presentation of the fetus E . Abdominal circumference
B . Level 1 ultrasound .
C . RH + typing of blood . * Methotrexate is ?
D . Blood pressure measurement . A . Anti_fungal agent
E . CBC . B . Anesthetic drug
C . Chelating agent
* One of the following is true about menopause ? D . Antimicrobial drug
A . Low LH and FSH level in blood . E . Antimetabolite drug
B . Normal estrogen and progesterone level in
blood . * All are causes of large for date uterus except ?
C . Smoking is protective from signs and A . Multiple gestation
symptoms . B . IUGR
D . Atrophy of the vagina and dyspareunia C . Wrong date
E . Median age is 40 years . D . Uterine fibroids
E . Infant of diabetic mother
* The incidence of breech presentation at term is
approximately ? * All are causes of small for date uterus except ?
A . < 1% A . Microsomia
B . <2% B . IUFD
C . <3 C . IUGR
D . <4% D . Wrong date
E . <5% E . Pregnancy with fibroid

* Most common cause of primary postpartum


hemorrhage is ?
A . Uterine atony
B . Retained placental tissue
C . Placenta accreta
* All of the following are benign structures D . Genital tract trauma
except E . DIC
A . Leiomyoma * One is false about insulin ?
B . Dermoid tumor A . Peptide hormone composed of 51 amino
C . Ovarian endometrioma acids .
D . Dysgerminoma B . Has catabolic effects
E . Uterine lipoma C . Released from B_cells of islets of Langerhans
from pancreas .
* Percentage of couple to be unable to conceive D . Requirements during pregnancy is increased
in first year of sexual life is approximately ? E . Increase glycogen synthesis in the liver .
A.5%
B . 10 % * First alarming sign of preeclampsia is ?
C . 15 % A . HTN
D . 20 % B . Proteinuria
E . 25 % C . Eclampsia
D . Edema
* The relation between long axis of the fetus to E . Visual changes
long axis of the mother is ?
A . Station * All are true about Fibronectin except ?
B . Attitude A . Positive Fibronectin test associated with
C . Position preterm labor .
D . Presentation B . It is high molecular weight glycoprotein
E . Lie C . Physiologically present after 20 weeks of
gestation
* Which of the following ultrasounds diameters D . It is produced by fetal cells
not used for fetal biometry ? E . It is adhesive or biologic glue that bind the
A . Biparietal diameter fetal sac to the uterus
B . Radial length
‫ اﺳﺎﻣﻪ اﻟﺨﺰاﻋﻠﻪ‬: ‫اﻟﺪﻛﺘﻮر‬ ‫اﺳﺌﻠﺔ اﻟﻨﺴﺎﺋﻴﺔ واﻟﺘﻮﻟﻴﺪ‬
Page 8
* True schedule of visits during pregnancy is ? A . Transverse vaginal septum .
A . First visit every month up to 26 weeks of B . Turner syndrome .
gestational age then 2 visits every month up to 36 C . Sheehan syndrome
weeks of gestational age weekly after 36 weeks D . Mullerian agenesis .
gestational E . Testicular feminization syndrome .
age
B . First visit every month up to 36 weeks * All are causes of secondary Amenorrhea
gestational age then weekly after 36 weeks except ?
gestational age . A . Pregnancy
C . 2 visits every month up to 26 weeks B . PCOS
gestational age then 3 visits every month up to C . Imperforate hymen
36 weeks gestational age then weekly after 36 D . Menopause
weeks gestational age . E . Prolactinoma
D . Weekly throughout whole pregnancy .
* Primary Amenorrhea is ?
* One is false about preterm labor ? A . Absence of menses at the age of 18 .
A . Presence of uterine contractions . B . Absence of menses for 6 months in women
B . Gestational age less than 37 weeks . who have previous normal menses .
C.Transvaginal ultrasound is contraindicated C . Absence of menses at the age of 16
D . Most common cause is preterm ROM . D . Absence of menses for 12 months in women
E . Positive Fibronectin test who have previous normal menses

* One of the following is not used for induction * Increasing of blood pressure in mild
of labour ? preeclampsia is up to ?
A . Ergometrine A . 149/99
B . Castor oil B . 169/109
C . Prostaglandin E2 C . 159/99
D . Cervical ripening digitally D . 159/109
E . Oxytocin infusion after ROM E . 149/109

* Amniotic fluid index in premature rupture of * About Anencephaly one of the following is true
membrane is approximately ? association ?
A.<5 A . It is compatible with life
B . < 10 B . Breech presentation
C . < 15 C . Face presentation
D . < 20 D . Oligohydramnios
E . < 25 E . Shoulder presentation

* Puerperium is ? * Incidence of congenital anomaly in infants of


A . Period from delivery of infants and placenta diabetic mother is approximately ?
to 7 weeks postpartum . A . 1_3%
B . Period from delivery of infants and placenta to B . 3_7 %
6 weeks postpartum C . 7_10%
C . Period from delivery of infants and placenta D . >10%
to 8 weeks postpartum . E . <1%
D . Period from delivery of infants and placenta
to 5 weeks postpartum * Most important criteria in assessment of first
stage of labour is ?
* Major risk factor for mastitis in Puerperium is ? A . Consistency of cervix
A . Herpes simplex type one . B . Position of the cervix
B . Staphylococcus aureus C . Maternal pulse and blood pressure
C . Candida albicans . D . Contractions of the uterus
D . Breast feeding . E . Cervical dilation
E . Cryptococcus
* Diagonal conjugate is equal ?
* All are causes of primary Amenorrhea except ? A . 10 cm
‫ اﺳﺎﻣﻪ اﻟﺨﺰاﻋﻠﻪ‬: ‫اﻟﺪﻛﺘﻮر‬ ‫اﺳﺌﻠﺔ اﻟﻨﺴﺎﺋﻴﺔ واﻟﺘﻮﻟﻴﺪ‬
Page 9
B . 8 cm * All of the following are true about drugs
C . 12 cm administration during pregnancy except ?
D . 14 cm A . ACE_i is contraindicated .
E . 15 cm B . Methotrexate is safe
C . CCB is safe .
* All are Malpresentation except ? D . Thyroxine is safe .
A . Breech E . Warfarin is contraindicated .
B . Shoulder
C . Face * About Abruption placenta , all are true except ?
D . Occipito_posterior A . Vaginal bleeding is present .
E . Compound B . Painful sensation .
C . Cesarean section is preferred method of
* Folic acid administration to prevent Neural management
tube defect , should be started ? D . Hard tender uterus .
A . Pre_conception E . Fetal distress up to fetal death .
B . At the end of first trimester
C . At the end of second trimester * One of the following is absolute
D . At the end of third trimester contraindication for HRT ?
E . At the 6 weeks of gestational age A . Personal history of breast cancer
B . Gallbladder disease
* All are true about endometriosis except ? C . Depression
A . Menorrhagia D . Fluid retention
B . Infertility E . Migraine
C . Most common site is `ovaries
D . No Dysmenorrhea * About anticholinergic drugs , all are true except
E . Surgery is the definitive treatment A . Used for treatment Bradycardia .
B . It has anti_sialagogue effect
* Most common of the following pathology is C . It is contraindicated in epilepsy
seen in clinical practice of gynecologist is ? D . It cause ataxia as a side effect
A . Endometrial cancer E . It is used for treatment urge incontinence
B . Ovarian cysts
C . Fibroids uterine
D . Ovarian Dysgerminoma
* Which of the following contraceptive is not * The major risk factor to gonorrhea in third
used during lactation ? trimester of pregnancy is ?
A . Injectable progesterone A .Polyarthritis
B . Implantable progesterone B .Postpartum endometritis
C . Mini pills C .Bartholin abscess
D . IUCD D .Tubo_ovarian abscess
E . COCP E .Ophthalmia neonatorum

* First step to do in female in reproductive years * Most common sexually transmitted disease
come to emergency room with acute abdominal is ?
pain is ? A .Syphilis
A .Abdominal ultrasound . B .Gonorrhea
B .Electrocardiogram . C .Chlamydia
C .Abdominal CT scan . D .Trichomonas
D .B_HCG E .Herpes simplex
E .Abdominal X_Ray .
* The major gas component that causes green
* The best test for diagnosis of hyperthyroidism house phenomenon is ?
during pregnancy is ? A .N2o
A .Free T4 B .Ozone
B .Total T4 C .Methane
C .Thyroid binding globulin D .CO2
D .Thyroid realising hormone E .CO
E .Total T3

‫ اﺳﺎﻣﻪ اﻟﺨﺰاﻋﻠﻪ‬: ‫اﻟﺪﻛﺘﻮر‬ ‫اﺳﺌﻠﺔ اﻟﻨﺴﺎﺋﻴﺔ واﻟﺘﻮﻟﻴﺪ‬


Page 10
* All are represented as descriptive type C .Preeclampsia
epidemiology except ? D .Smoking
A . Why ? E .Oligohydramnios
B . Who ?
C . What ? * About ectopic pregnancy all are true except ?
D . When ? A . Clomephene citrate is risk factor .
E . Where ? B . Associated with uterine enlargement .
C . It's located in the ovary in 50% .
* Incidence is ? D . Ectopic pregnancy with serum B_HCG < 1000
A . The number of old cases , episodes or events ML U/ L can be . Treated conservative .
that occurred over defined period of time . E . Diagnostic laparoscopy has a great value in
B . The number of new cases episodes or events management .
that occur over defined period of time
C . The sum of new and old cases , episodes, or * 30 years old woman is known to have
events that occur over defined period of time . Antiphospholipid syndrome , all are true except ?
D . The number of death's occurring over a A . Her risk of DVT is increased .
defined period of time . B . Her risk of abortion is reduced by steroids .
C . 50% associated with SLE .
* Prevalence is ? D . OCP is absolute contraindicated .
A . The number of new cases episodes or events E . Prolonged PTT .
occur over a defined period of time .
B . The number of old cases episodes or events * The best most sensitive screening test for
occur over a defined period of time . gestational diabetes mellitus ?
C . The number of death's occur over a defined A . Glucose in urine .
period of time . B . Random glucose measurement .
D . The total number of existing cases , episodes C . Fasting glucose measurement .
or events occurring at one point in time D . HbA1c .
E . One hour glucose tolerance test .
* All are true statement except ?
A . HIV virus is an RNA retrovirus .
B . Peak incidence of HIV - 15_25 years old .
C . Condom always protect from sexual
transmitted diseases
D . The best way to prevent STDs is abstinence .
E . Most STDs are asymptomatic .
* Ozone layer protects us from ?
A . Infrared radiation * All are true about anemia in pregnancy except
B . Ultraviolet radiation A . It's associated with multiparity .
C . Gamma radiation B . MCV is increased in thalassemia .
D . Heavy metal radiation C . Increased lobes of Nuclei of Neutrophils
E . Alpha and beta radiation associated with folate deficiency .
D . Vitamin B12 deficiency is associated with
* About molar pregnancy , all are true except ? Megaloblastic picture in bone marrow .
A . Incomplete mole mostly have 69 xxy E . Fetal hydrops is a complication of fetus with
karyotype . alpha thalassemia
B . Complete mole associated with ovarian theca
cysts . * The following conditions are worsening during
C . Women must be advised not to conceive until pregnancy except ?
HCG levels have return to normal . A .Sickle cell disease
D . Medical evacuation using PGE2 and oxytocin B .Eisenmenger syndrome
is the treatment of choice . C .Marfan syndrome
E . It's benign tumor , but there is malignant D .Mitral stenosis
potential E .Irritable bowel syndrome

* All are risk factors for placental Abruption * All are components of Meigs syndrome except
except ? A .Amenorrhea
A .Polyhydramnios B .Pelvic mass
B .External cephalic version C .Ascites
‫ اﺳﺎﻣﻪ اﻟﺨﺰاﻋﻠﻪ‬: ‫اﻟﺪﻛﺘﻮر‬ ‫اﺳﺌﻠﺔ اﻟﻨﺴﺎﺋﻴﺔ واﻟﺘﻮﻟﻴﺪ‬
Page 11
D .Pleural effusion C . Late deceleration .
D . Fetal Intraventricular hemorrhage .
* Estrogen secreting ovarian tumor include ? E . Decreased variability on CTG .
A. Choriocarcinoma
B .Yolk sac tumor * All are true about oxytocin except ?
C .Dysgerminoma A . It's Glycoprotein hormone .
D .Serous cystadenoma B. It's secreted from posterior pituitary .
E .Granulosa cell tumor C . It has an antidiuretic effect .
D . Produces uterine spasms in usually doses .
* All are absolute contraindications to COCPs E . Most common side effect is hypotension and
except ? tachycardia
Migraine with aura
History of DVT * All of the following drugs have a tocolytic
Severe DM ‫اﻻﺟﺎﺑﺘﻴﻦ ﺻﺤﻴﺤﺘﻴﻦ‬ effect except ?
Aspirin
Lactation
Ethanol
Active liver disease
Nimesulide
Atosiban
* About gestational diabetes mellitus , all are
Terbutalin
true except ?
A . Congenital anomalies are less common .
* All are side effects of Mgso4 except ?
B . Neonatal hyperglycemia is uncommon .
Tachypnoea
C . It's secondary to insulin resistance .
Flushing of skin
D . Maternal insulin requirements increase after
Wide QRS complex
delivery .
Nausea and vomiting
Atonic baby syndrome
* HCG is useful in monitoring one of the
following tumors ?
* All are true about endometriosis except ?
Yolk sac tumors
A . Most common presentation is Menorrhagia .
Ovarian trratoma
B . It's recognized cause of infertility .
Persistent invasive mole
C . Commonest site in ovaries .
Granulosa cell tumors
D . Dysmenorrhea is common symptom .
Struma ovarii
E . Definitive treatment is hysterectomy .

* All are true about chlamydia infection in


* All are true about Rubella infection in
pregnancy except ?
pregnancy except ?
A . It's gram positive intracellular organism .
A . Occurring 8 weeks gestation results in high
B . It's sensitive to tetracycline .
rate of fetal infection .
C . Commonest bacterial sexual infection .
B . It's associated with congenital heart disease .
D . Best treatment during pregnancy is
C . Detection of rubella IgG in cord blood confirm
Macrolides .
fetal infection .
E . Causes pelvic inflammatory disease and
D . It's associated with IUGR .
lymphogranuloma venereum .
E . Detection of rubella IgM in maternal blood
confirm maternal infection .
* All are indications for delivary in PET except ?
Eclampsia
* All are risk factors for osteoporosis except ?
HELLP syndrome
Alcohol abuse
HTN uncontrollable
Smoking
Acute renal failure
Corticosteroids
Protein in urine 3 g/day
Hyperthyroidism
High BMI
* Fetal acidosis in Labour is associated with all
of the following except ?
* All are true about Tamoxifen except ?
A . Prolong uterine hypertonus .
A . It's used for treatment and prophylaxis breast
B . Prolong pregnancy .
cancer .
‫ اﺳﺎﻣﻪ اﻟﺨﺰاﻋﻠﻪ‬: ‫اﻟﺪﻛﺘﻮر‬ ‫اﺳﺌﻠﺔ اﻟﻨﺴﺎﺋﻴﺔ واﻟﺘﻮﻟﻴﺪ‬
Page 12
B . It has estrogenic effect on endometrium . Sheehan syndrome
C . Is associated with Amenorrhea . Prolactinoma
D . Decreasing the risk of DVT and PE . Pituitary failure

* 16 year old female with primary Amenorrhea * All are true clinical presentation of PCOS
and raised LH and FSH level , diagnosis is ? except ?
A .Turner syndrome Irregular uterine bleeding
B .Kallmans syndrome Low BMI
C .Rokitanski syndrome Hirsutism
D .Stein_Leventhal syndrome Acanthosis nigricans
E .Sheehan syndrome Hyperandrogenism

* All are true about polycystic ovary syndrome * In a pregnant woman over the age of 35 years
except ? there is a risk of all except ?
A . Associated with LH Hypersecretion and FSH Ectopic pregnancy
Hypersecretion . Monozygotic twins
B . Typical cause of secondary Amenorrhea . Trophoblastic disease
C . Hypersecretion of DHEAS from adrenal gland Preterm delivery
D . It is a uncommon cause of anovulation . Down syndrome
E . It is a recognized cause of infertility .
* The following are complications of babies for
* About brachial plexus injury , all are true diabetic mothers except ?
except ? A .Brachial plexus injury
A . Erbs palsy is associated with C5_C6 injury . B .Neonatal hyperglycemia
B . Klumpkes palsy is associated with C7_C8 C .RDS for newborn
injury . D .Jaundice
C . Erbs palsy is associated with absent moro E .Neonatal Hypocalcemia
reflex .
D . Klumpkes palsy is associated with Horner * All are risk factors for endometrial cancer
syndrome . except ?
E . Physiotherapy is the main mode of treatment . A . Polycystic ovary syndrome .
B . DM .
C . HTN .
D . Ovarian Dysgerminoma .
E .Family history of breast or colon cancer .

* All are true about congenital adrenal * BRCA1 and BRCA2 mutations associated with
hyperplasia except ? all of the following malignancy except ?
A . Only female fetus are affected . A .Lung cancer
B . 21 _ ( OH ) case deficiency is the commonest B .Breast cancer
C . It's associated with clitormegaly and C .Colorectal cancer
ambiguous genitalia . D .Prostatic cancer
D . Uterus , fallop tubes , vagina are present . E .Ovarian cancer
E . Raised ACTH , 17 ( OH ) progesterone .
* Oligohydramnios is associated with all
* Development dysplasia of the hip , all are true except ?
except ? A . Twin _ Twin Transfusion syndrome .
A . It's more common in first born female . B . Antenatal use of NSAIDs .
B . More common in Oligohydramnios . C . Neural tube defect .
C . It screened using Barlow and Ortolani tests . D . Post _ maturity
D . Affect the right hip more than left hip . E . Potter syndrome
E . More common in breech presentation .
* All are risk factors for IUFD except ?
* 30 years old female with secondary A .External cephalic version
Amenorrhea and low gonadotropines level , all B .Chromosomal anomalies
are true except ? C .TORCH infection
Premature ovarian failure D.Twin Twin Transfusion syndrome
‫ اﺳﺎﻣﻪ اﻟﺨﺰاﻋﻠﻪ‬: ‫اﻟﺪﻛﺘﻮر‬ ‫اﺳﺌﻠﺔ اﻟﻨﺴﺎﺋﻴﺔ واﻟﺘﻮﻟﻴﺪ‬
Page 13
E .Bacterial vaginosis * All of the following are associated with high
risk of maternal mortality except ?
* A 30 years old woman is found to have a A .Pulmonary HTN
hydrops fetalis 25 weeks gestation , the following B .ASD
are possible causes except ? C .Marfan syndrome
A . Parvovirus B19 infection . D . HOCM
B . B thalassemia . E .Postpartum Sheehan syndrome
C . RH_isoimmunization .
D . STORH infection . * All of the following are causes of secondary
E . Chromosomal anomalies . Amenorrhea except ?
A .Cushing syndrome
* All are true about Turner syndrome except ? B .Hypothyroidism
A . Impaired intellectual function . C .Congenital adrenal hyperplasia
B . Streak gonads with primary Amenorrhea . D .Asherman syndrome
C . Associated with fetal hydrops and cystic E .Kallmans syndrome
hygroma .
D . No increase risk of ovarian cancer . * All of the following are associated with
E . Coarctation of the aorta is common . hyperprolactinemia except ?
A .Cushing syndrome
* Long term sequelae of PCOS include all of the B .Cushing disease
following except ? C .Acromegaly
A .DM type two D .Primary hypothyroidism
B. HTN E .PCOS
C .Cervical cancer
D. Infertility * All are true about Hematocolpos except ?
E .High BMI A . Associated with secondary Amenorrhea .
B . Presents with cyclic lower abdominal pain .
* All of the following are side effects of C . Presents with acute urine retention .
Clomephene citrate except ? D . On PR exam _ mass in vagina .
A .Menorrhagia E . Patient has normal vagina , uterus , tubes and
B. Amenorrhea ovaries
C .Ovarian hyperstimulation syndrome
D .Breast tenderness * In Neonate with ambiguous genitalia , all of the
E .Hair loss and hot flushes following are true except ?
A . Karyotype is mandatory .
B . Turner syndrome is a cause .
C . Electrolytes are mandatory .
D . Congenital adrenal hyperplasia is a cause .
E . 5_alpha reductase deficiency is a cause .
* All are associated with worse prognosis in a
woman with Choriocarcinoma except ? * All of the following are features of septic
A . Age > 40 years old . abortion except ?
B . Choriocarcinoma after a normal term A .DIC
pregnancy . B .Jaundice
C . Multipara woman ( > 4 pregnancy ) . C .HTN
D. Blood group O and A . D .Acute renal failure
E . Long interval between pregnancy and E .Hyper or hypothermia
chemotherapy > 4 months .
* All are true regarding placenta previa except ?
* All of the following are causes of raised A . Antepartum blood loss is fetal blood .
MSAFP except ? B . Associated with an increased risk of placenta
A .Multiple pregnancy accreta .
B .Choriocarcinoma C . Associated with vaginal bleeding in the third
C .Antipartum hemorrhage trimester .
D .Down syndrome D . Painless massive vaginal bleeding without
E .Sacro_coccygeal trratoma fetal distress .

‫ اﺳﺎﻣﻪ اﻟﺨﺰاﻋﻠﻪ‬: ‫اﻟﺪﻛﺘﻮر‬ ‫اﺳﺌﻠﺔ اﻟﻨﺴﺎﺋﻴﺔ واﻟﺘﻮﻟﻴﺪ‬


Page 14
* A 34 years old female presents a 34 weeks * About ovarian Dysgerminoma . One is true ?
gestation with vaginal bleeding and abdominal A . Associated with Hirsutism .
pain , her blood pressure 150/90 , fetal heart rate B . Typically bilateral .
170 bmp , all of the following are true except ? C . Typically radioresistant .
A . Most likely diagnosis is Abruption placenta . D . Sex cords derived tumor .
B . Emergency cesarean section must be done if E . Peak age > 40 years old
fetal heart rate deceleration are present .
C . Thrombocytosis is a complication . * All are absolute contraindications for IUCD
D . Acute renal failure is a complication . except ?
E . Central venous pressure is indicated . A . Recent STDs or PID .
B . Undiagnosed vaginal bleeding .
* All of the following associated with an C . Wilson disease .
increased risk of first trimester abortion except ? D . Past history of ectopic pregnancy .
A .PCOS E . Pregnancy .
B. Chromosomal anomalies
C .Luteal phase defect * All are risk factors for vaginal Candida except ?
D .Methotrexate usage A . DM
E .Well controlled DM type two B . Pregnancy
C . COCP
* All are risk factors for placental previa except ? D . IUCD
A .Teenage pregnancy E . Broad spectrum antibiotics therapy .
B .Previous uterine infection
C .Previous scar * Strawberry cervix is a classical sign of ?
D .Multiparity A .Bacterial vaginosis
E .Smoking B .Trichomonas vaginalis
C .Chlamydia trachomatis
* About gestational diabetes mellitus all are true D .Vaginal Candidiasis
except? E .Troponema pallidum
A . Neonatal jaundice is more common .
B . Neonatal Hypocalcemia is complication . * About MIRENA , all are true except ?
C . Insulin requirement increased during A . It's contraindicated in Wilson disease .
pregnancy and fall after delivery . B . Used as an IUCD for 5 years .
D . Induction of labour in well controlled GDM at C . Used for treatment Menorrhagia .
37 weeks gestation . D . Contains 52 mg levonorgestrel , which
E . Obesity , old age , family history are the major released at the rate 20 mcg / day .
risk factors . E . It's very effective for contraception and has
very low failure rate .

* About hyperemesis gravidarum all are true * All of the following are side effects of
except ? Nifedipine in treatment preeclampsia except ?
A . Associated with metabolic alkalosis . A .Gum hyperplasia
B . Associated with HTN . B .Gynecomastia
C . Associated with Hypokalemia , C .Bradycardia
Hypochloremia . D .Ankle edema
D . It's uncommon after 16 weeks . E .Face flushing
E . Urine is acidic .
* With regards to the anatomy and mechanism
* One of the following is skin manifestation of of labour , all are true except ?
normal pregnancy ? A . The sacral promontory is the posterior
A .Spider nevi landmark of the pelvic inlet .
B .Pemphigoid B . Ischial spines are the lateral landmarks of the
C .Vitiligo mid pelvis .
D .Erythema herpetiforms C . Symphysis pubis and body of pubic bone are .
E .Pyoderma gangrenosum The anterior landmarks of the pelvic inlet

‫ اﺳﺎﻣﻪ اﻟﺨﺰاﻋﻠﻪ‬: ‫اﻟﺪﻛﺘﻮر‬ ‫اﺳﺌﻠﺔ اﻟﻨﺴﺎﺋﻴﺔ واﻟﺘﻮﻟﻴﺪ‬


Page 15
D . The transverse diameter of the pelvic inlet is A . Monozygotic twins have the same incidence
the widest diameter and equal 13 cm . worldwide .
E . The anterior _ posterior diameter of the pelvic B . Dizygotic twins are more common .
outlet is the narrowest diameter and equal 11 cm C . Monozygotic twins are always identical .
D . Cesarean section is recommended if the first
* All are true about late deceleration except ? twin is breech presentation .
A . Reach their minimum after the peak of the E . Cesarean section is recommended if the
contraction . second twin is breech presentation .
B . Associated with uteroplacental failure .
C . It's indicate fetal distress . * All are relative contraindication to use
D . It's always associated with cord PH < 7.2 antenatal corticosteroids except ?
E . Seen in severe preeclampsia . A .Multiple pregnancy
B .Maternal DM
* All are risk factors for PPH except ? C .Chorioamnionitis
A . Multiparity D .Term pregnancy
B . Active management of third stage of labour .
C . Rapid and prolonged labour .
D . General anaesthesia * Most common cause of postmenopausal
E . Coagulopathy bleeding ?
A . Cervical cancer
* There is increased risk of all of the following in B . Endometrial cancer
the post partum compared with Antepartum C . Cervical polyps
except ? D . Atrophic vaginitis
A . Blues , depression , psychosis . E . Endometriosis
B . Thromboembolism , DVT , PE .
C . Milk stasis and mastitis . * 20 years old pregnant lady , 21 week
D . Insulin resistance and hyperglycemia . gestational age , was done for her ultrasound .
E . High cardiac output heart failure . Found to have low lying placenta , what must you
do ?
* Fetal Hydrocephalus associated with ? A . Do weekly ultrasound .
A. Arnold Chiari malformation B . Do at 28 weeks ultrasound .
B .Fragile x syndrome C . Do pelvic MRI not at 21 week .
C .Achondroplasia D . Do at 25 week ultrasound .
D .Turner syndrome E . No need for follow up .
E .VWD
* Most significant serious complication for PET
* Amniocentesis is useful in diagnosis all of the and eclampsia is ?
following except ? A . HELLP syndrome
A .Fragile x syndrome B . Acute renal failure
B .CAH C . Cerebral hemorrhage
C .Spina bifida D . Blindness
D .Lung maturity E . Pulmonary edema
E .Esophageal atresia
* All are true matching about head and
presentation except ? * All are true about HCG except ?
A. Vertex : suboccipito _ bregmatic 9.5 cm . A . Primarily produced by syncytiotrophoblasts .
B . Military : occipitofrontal 11 cm . B . Detected in serum from 10 days after
C . Brow : occipitomental 12 cm . fertilization .
D . Face : submento_bregmatic 9.5 cm . C . Reach the peak at 10 weeks .
D . It is glycoprotein hormone .
* Polyhydramnios is associated with all ,except ? E . It has very high level in Choriocarcinoma .
A . Imperforate anus
B . Congenital Diaphragmatic hernia . * All are component of posterior pelvis except ?
C . Chromosomal anomalies . A . Sacrum
D . Twin Twin Transfusion syndrome . B . Coccyx
E . Post maturity C . Urethral orifice
D . Rectum
* Regarding twin pregnancy , all are true except E . Sigmoid
‫ اﺳﺎﻣﻪ اﻟﺨﺰاﻋﻠﻪ‬: ‫اﻟﺪﻛﺘﻮر‬ ‫اﺳﺌﻠﺔ اﻟﻨﺴﺎﺋﻴﺔ واﻟﺘﻮﻟﻴﺪ‬
Page 16
* Cause of Gonadectomy after puberty in
* All of the following are causes of candidiasis management of Androgen insensitivity syndrome
except ? is ?
A . Pregnancy A . Risk of gynecomastia
B . Antibiotics B . Risk of breast cancer
C . OCPs C . Risk of testicular cancer
D . Hypoglycemia D . Risk of liver cancer
E . Steroids E . Risk of ovarian cancer

* Treatment for mild cases of PID is ? * About infertility , all are true except ?
A . Ofloxacin and metronidazole A . PCOS is the most common endocrine cause
B . Penicillin and metronidazole in women .
C . Rifampin and Streptomycin B . Adenomyosis is recognized cause .
D . Clindamycin and Ampicillin C . Unexplained cause about 15_30%
E . Ofloxacin and vancomycin D . Semen analysis must be done early .
E . Kallmans syndrome is one of the causes .
* All are true about treatment of cervical cancer
except ? * All are true criteria for vacuum extraction
A . Management depends on stage and age . vaginal delivery except ?
B . Stage 1a treated by combination chemo_radio A . Experience operator
C . Stage 2a treated mainly chemo_radio . B . Fully dilated cervix
D . Proceed with Wertheims hysterectomy if C . More than 30 weeks gestation
lymph nodes negative . D . Head presentation and vertex attitude
E . Stage 0 treated by LEEP E . Rupture of membranes

* Pregnant women gestational age 18 week and * The type of presentation when the baby's
in physical exam fundus of uterus buttocks lie over the maternal pelvis , and flexed
above the umbilicus , all are causes except ? hip and extended knee is called ?
A . Gestational diabetes mellitus . A . Footling breech
B . Duodenal atresia . B . Frank breech
C . STORH infection . C . Flexed breech
D . Fetal hydrops . D . Combined breech
E . NTD like Anencephaly . E . Cephalic

* All are true about epilepsy management in * The type of presentation when the baby's
pregnancy except ? buttocks lie over the maternal pelvis and flexed
A . Valproate can cause NTD 10 folds . both hips and knees is called ?
B . Phenytoin can cause cardiac anomalies . A . Footling breech
C . Lamotrigine can cause cleft lip and palate B . Frank breech
D . There is an increased risk of major congenital C . Complete breech
anomalies 5_10% D . Compined breech
E . Folic acid 5 mg reduces the risk of some E . Cephalic
anomalies .

* All are effect of diabetes in pregnancy except ?


A . Abortion * All are causes of breech presentation except ?
B . Macrocephaly fetal A . Fibroids
C . Macrosomia fetal B . Multiple pregnancy
D . Neonatal cardiomegaly C . Post_term
E . Neonatal Hyaline membrane disease D . Polyhydramnios
E . Fetal abnormalities
* The LH surge occurs before ovulation by ?
A . 6 hours * All are risk factors for osteoporosis except ?
B . 8 hours A . Smoking
C . 12_36 hours B . Rheumatoid arthritis
D . 72 hours C . Hypogonadism
E . 36_72 hours D . Hypoparathyroidism
E . Low BMI
‫ اﺳﺎﻣﻪ اﻟﺨﺰاﻋﻠﻪ‬: ‫اﻟﺪﻛﺘﻮر‬ ‫اﺳﺌﻠﺔ اﻟﻨﺴﺎﺋﻴﺔ واﻟﺘﻮﻟﻴﺪ‬
Page 17
C . Obesity
* Most common cause of Abruption placenta D . Endometrial cancer
is ? E . Ovarian cancer
A . Trauma
B . Previous scar * Menstrual disorder in Asherman syndrome is ?
C . Preeclampsia A . Hypomenorrhea
D . Idiopathic B . Menorrhagia
E . Multiple pregnancy C . Metrorrhagia
D . Oligomenorrhea
* All are true about placenta previa except ? E . Dysmenorrhea
A . Vaginal exam is contraindicated .
B . Lower uterine segment placenta implantation * All are causes of Oligohydramnios except ?
C . About 0.5% of pregnancies at term A . Tracheo_esophageal fistula fetal
D . Painful third trimester vaginal bleeding B . IUGR
E . Delivery is likely to be by cesarean section C . Renal agenesis
D . Post date pregnancy
* All are side effects of COCP except ? E . Posterior urethral valve
A . Breakthrough bleeding
B . Headache * Variable deceleration on CTG means ?
C . Increased risk of stroke A . Utero_placental insufficiency
D . Increased risk of ovarian cancer B . IUGR
E . HTN and hyperglycemia C . Head compression
D . Umbilical cord compression
* All are contraindications for IUCD except ? E . Fetal movement
A . Vaginal bleeding
B . Active pelvic inflammatory disease * All are present in Bishop score to assessment
C . Copper allergy the favorability for induction of labour except ?
D . Uterine anomalies A . Cervical dilation
E . >35 years old smoker B . Cervical consistency
C . Cervical length
* Menopause is ? D . Cervical position
A . Permanent 12 consecutive months of E . Cervical station
Amenorrhea that result from loss of hypophysis
activity . * All are true about first stage of labour except ?
B . Permanent 12 consecutive months of A . It is divided into two phases .
Amenorrhea that result from loss of uterine B . Progressive cervical dilation .
activity . C . Fetal heart rate must be monitored every 15
C . Permanent 6 consecutive months of min .
Amenorrhea that result from loss of ovaries D . Vaginal exam must be offered every 4 hours
activity . to assess progress of labour .
D . Permanent 12 consecutive months of E . IUGR causes poor progress in the first stage .
Amenorrhea that result from loss of ovaries
activity * All of the following are causes of DVT except
A . SLE
B . Nulliparity
C . SCA
D . PRV
* Average age of menopause in Jordan is ? E . Antiphospholipid syndrome
A . 45 years old
B . 51 years old * Active management of the third stage of labour
C . 55 years old include all of the following except ?
D . 60 years old A . Use of oxytocin
E . 61 years old B . Use of Methylergonovine
C . Controlled cord traction
* All are long term consequences of PCOS D . Use of progesterone
except ? E . Early clamping and cutting of the cord .
A . Endometrial hyperplasia
B . Insulin resistance
‫ اﺳﺎﻣﻪ اﻟﺨﺰاﻋﻠﻪ‬: ‫اﻟﺪﻛﺘﻮر‬ ‫اﺳﺌﻠﺔ اﻟﻨﺴﺎﺋﻴﺔ واﻟﺘﻮﻟﻴﺪ‬
Page 18
* All are increased during normal pregnancy * All are true about premature ovarian failure
except ? except ?
A . Cardiac output A . Chromosomal abnormalities are causes .
B . Heart rate B . Most common presentation is secondary
C . Red blood cell mass Amenorrhea .
D . Systemic vascular resistance C . Women need progesterone therapy until 52
E . Plasma volume years old .
D . At increased risk of osteoporosis .
* All of the following are increased in normal E . At lower risk of breast cancer .
pregnancy except ?
A . Factor 1 * All are sexual transmitted diseases except ?
B . Factor 7 A . Bacterial vaginosis
C . Factor 8 B . Chlamydia infection
D . VWF C . Trachomatis infection
E . Fibrinolysis D . Gonococcus infection
E . HPV
* All are true about normal finding in urine
analysis in normal pregnancy except ? * Most important ligament that give support to
A . Proteinuria uterus is ?
B . Glucosuria A . Uterosacral ligament
C . Hematuria B . Cardinal ligament
D . Alkaline urine C . Broad ligament
E . Ketonuria D . Ovarian ligament
E . Pubocervical ligament
* About molar pregnancy . One is false ?
A . Complete mole consists of diffuse hydropic * Where we can hear fetal heart sound in breech
villi with Trophoblastic hyperplasia . presentation ?
B . Partial mole associated with no fetal tissue . A . At or above the level of the maternal
C . Most common symptom is irregular first umbilicus .
trimester vaginal bleeding . B . At or below the level of the maternal
D . Associated with hyperemesis and umbilicus
hyperthyroidism . C . At or above the level of the pubic bone of
E . Snow storm appearance on ultrasound for mother .
complete mole . D . At or above the level of the anterior shoulder
of the fetu
* Choriocarcinoma spread mainly by ?
A . Lymphatic * All are side effects of MIRENA IUD except ?
B . Blood A . Menorrhagia
C . Direct B . Irregular P.V bleeding
D . Transcoelomic C . Nausea
E . Peritoneal D . Breast tenderness
E . Bloating
* Side effects injectable progesterone are all of
the following except ? * Drug of choice in HTN in pregnancy is ?
A . Menstrual disturbance A . Methyldopa
B . Delayed conception for 6_12 months B . Sulfate magnesium
C . Failure rate 2% which is high C . Thiazide
D . Weight gain D . Hydralazine
E . Bone loss with long use E . Labetalol
* 25 year old female , pregnant first time with
twins , no past history of abortion , one is correct * Screening tool for Down syndrome in 17 weeks
regarding this pregnant ? gestation is ?
A . G2 P2 A . Level 1 ultrasound
B . G1 P1 B . Level 2 ultrasound
C . G2 P1 C . Double screening test
D . G1 P0 D . Triple screening test
E . G0 P1 E . No screening at 17 weeks

‫ اﺳﺎﻣﻪ اﻟﺨﺰاﻋﻠﻪ‬: ‫اﻟﺪﻛﺘﻮر‬ ‫اﺳﺌﻠﺔ اﻟﻨﺴﺎﺋﻴﺔ واﻟﺘﻮﻟﻴﺪ‬


Page 19
* Diagnosis of choice for ectopic pregnancy is ? * All of the following are decreased during
A . HSG normal pregnancy except ?
B . Hysteroscopy A . Hb and hematocrit
C . Laparoscopy B . Platelet count
D . CT scan C . SVR
E . MRI D . Respiratory rate
E . Serum uric acid
* Most important symptom of ectopic pregnancy
A . Vaginal bleeding * All are early symptom of pregnancy except ?
B . Abdominal pain A . Morning sickness
C . Amenorrhea B . Amenorrhea
D . Increased size of abdomen C . Pica
E . Shoulder tip pain D . Breast tenderness
E . Quickening
* Absolute contraindication for OCP is ?
A . Gallbladder stones * All are risk factors for vasa previa except ?
B . Lactating mother A . Multiple pregnancy
C . Migraine without aura B . High lying placenta
D . Controlled DM or HTN C . Bilobed placenta
E . Age >35 , smoking mother D . Succenturiate placenta
E . IVF pregnancy
* Treatment of choice for Gonococcal PID is ?
A . Ceftriaxone * All are risk factors for PET except ?
B . Metronidazole A . Past history
C . Fluconazole B . Extreme of ages
D . Amoxicillin C . Obesity
E . Ofloxacin D . Multiparity
E . Hydatiform mole
* The gold standard for diagnosis PID is ?
A . Ultrasound * All are increased in PET except ?
B . Laparoscopy A . Hb and hematocrit
C . HSG B . Platelet
D . Hysteroscopy C . PT and PTT
E . Cervical and vaginal swab culture D . Uric acid
E . ALT and AST
* All are true about physiology of pregnancy
except ? * Most common symptom of HELLP syndrome ?
A . Decreased in Hb and hematocrit A . Nausea
B . Anemia is diagnosed if Hb <9.5 g/dl B . Vomiting
C . No change in MCV , MCH , MCHC in normal C . HTN and headache
pregnancy . D . Epigastric or RUQ pain
D . Normally 2_3 fold increase iron requirement E . Purpura
E . Normally 10_20 fold increase folate
requirement * Most common typical timing of division in
Monozygotic twins is ?
* The largest diameter of fetal head is ? A . < 3 days
A . Mento_vertical B . 4_7 days
B . Submento_bregmatic C . 8_12 days
C . Occipitofrontal D . 12_21 days
D . Suboccipito_frontal E . > 21 days
E . Suboccipito_bregmatic
* In full term fetus the blood flow in the cord ?
A . 150 ml/min * Normal CTG include all except ?
B . 200 ml/min A . Baseline fetal heart rate 110_160 BPM .
C . 250 ml/min B . No variability
D . 350 ml/min C . Two acceleration in 20 minutes
E . 450 ml/min D . Absent deceleration
E . Absent sinusoidal fetal heart rate rhythm
‫ اﺳﺎﻣﻪ اﻟﺨﺰاﻋﻠﻪ‬: ‫اﻟﺪﻛﺘﻮر‬ ‫اﺳﺌﻠﺔ اﻟﻨﺴﺎﺋﻴﺔ واﻟﺘﻮﻟﻴﺪ‬
Page 20
E . Second pregnancy .
* Most common cause of pneumonia in
pregnancy ? * All of the following are prerequisites for the
A . Haemophilus influenza application of instrumental delivery except ?
B . Staphylococcus aureus A . The cervix is dilated 9 cm .
C . Klebsiella B . Maternal bladder is empty .
D . Streptococcus pneumonia C . The fetal head is engaged .
E . Mycoplasma pneumonia D . The fetal position is known .
E . The mother has adequate analgesia .
* Causes of hyperprolactinemia all , except ?
A . Pregnancy * The commonest type of uterovaginal prolapse
B . Empty sella syndrome A . Cytocele .
C . Hyperthyroidism B . Rectocele .
D . Chronic renal failure C . Enterocele .
E . Methyldopa D . Uterine prolapse .
E . Urethrocele .
* Long term effect of recurrent cesarean section
are all of the following except ? * The best independent predictor of induction of
A . Uterine rupture labor success at the time ?
B . Placenta previa A . Cervical position .
C . Placenta Abruption B . Cervical effacement .
D . Placenta accreta C . Cervical consistency .
E . Placenta inccreta D . Station of the head .
E . Cervical dilation .
* All are true about management of ovarian cyst
in pregnancy except ? * Endometriosis is associated with all of the
A . Asymptomatic , non_enlarge cyst must be following except ?
managed surgically . A . Chocolate cyst .
B . Cystectomy is performed in symptomatic B . Infertility .
acute complication , malignancy , large in size > 8 C . Ectopic pregnancy .
cm D . Dysmenorrhea .
C. Elective surgery at 16_20 weeks . E . Late menopause .
D . If cyst causes obstruction of labour , delivery
must be cesarean section * All of the following are indications for induction
E . Increase risk of miscarriage or preterm of labor except ?
delivery A . Prolonged pregnancy .
B . IUGR .
* The most common pelvic shape in the females C . Abnormal lie .
is ? D . Gestational HTN .
A . The Gynecoid . E . Term decreased fetal movement .
B . The android .
C . The anthropoid . * All of the following may be found in ovarian
D . The platypelloid . hyperstimulation syndrome except ?
A . Large ovaries bilateral .
* During fetal monitoring in labor if there is cord B . Hydrothorax may develop .
compression this is usually represented as ? C . Ascites may develop .
A . Recurrent variable deceleration . D . Hypovolemia develop .
B . Absent baseline variability . E . Hyperproteinemia may develop .
C . Recurrent late decelerations .
D . Prolonged Bradycardia .
E . Sinusoidal patterns .

* All of the following are different causes of * The best contraceptive method in 40 years old
preterm labor except ? woman is ?
A . Multiple gestations . A . COCP .
B . PROM . B . IUD .
C . Pregnancy associated HTN . C . POP .
D . Cervical incompetence . D . Hysterectomy .
‫ اﺳﺎﻣﻪ اﻟﺨﺰاﻋﻠﻪ‬: ‫اﻟﺪﻛﺘﻮر‬ ‫اﺳﺌﻠﺔ اﻟﻨﺴﺎﺋﻴﺔ واﻟﺘﻮﻟﻴﺪ‬
Page 21
E . The contraceptive implant . E . The presence of IUCD .

* All of the following are early symptom of * Total abdominal hysterectomy is accepted the
estrogen deficiency at the menopausal years procedure of choice in all of the following
except ? condition except ?
A . Irritability . A . Chronic pelvic inflammatory disease .
B . Insomnia . B . Uterine myomas .
C . Hot flushes . C . Stage 1 endometrial cancer .
D . Osteoporosis . D . Stage 1 cervical cancer .
E . Mood distrubances . E . Endometriosis .

* The first logic initial diagnostic test for acute * One of the following is false ?
excessive bleeding in women of reproductive age A . Early deceleration and fetal scalp
A . Pregnancy test . compression .
B . Pelvic Ultrasonography . B . Variable deceleration and umbilical cord
C . Endometrial biopsy . compression .
D . Complete blood count . C . Late deceleration and placental insufficiency .
E . Coagulation test . D . Sinusoidal fetal heart tracing pattern and fetal
anemia .
* The most distressing symptom during the E . Normal beat to beat and fetal hypoxia .
menopause years among the following is ?
A . Hot flushes * The elevation of basal body temperature just
B . Insomnia before ovulation is related mainly to ?
C . Irritability A . Central effect of progesterone .
D . Mood disturbances B . The luteinizing hormone surge .
E . Osteoporosis C . Effect of specific sex hormone binding
globulin .
* The detailed fetal anomaly scan is best D . Immunologic reaction effect of the pituitary
performed between which gestational age ? hormones .
A . 15 _ 18 weeks E . Transformation of follicular endometrium into
B . 18 _ 23 weeks secretory endometrium .
C . 23 _ 26 weeks
D. 26 _ 28 weeks * All of the following can cause Polyhydramnios
E . 28 _ 30 weeks except ?
A . IUGR .
* Leukemia can be caused by one of the B . Gestational diabetes .
following ? C . Spina bifida .
A . Toluene D . Skeletal dysplasia .
B . Tetrachloromethane E . Tracheosesophageal atresia
C . Acetone
D . Benzene * Regarding dysfunctional uterine bleeding , all
E . Ethyl lead of the following are true except ?
A . Is common after menarch .
* Which of the following vaccine is not given to B . Commonly occurs with polycystic ovarian
pregnant women ? syndrome .
A . Pertussis . C . Can be treated with copper releasing IUCD
B. Hepatitis B . D . Is commonly managed with hormonal therapy
C . Diphtheria . E . Is common in the perimenopausal period
D . Rubella .
E . Influenza .

* All of the following can cause secondary * When using oxytocin for induction or
dysmenorrhea except ? augmentation of labor , we stop increasing it
A . Fibroids . when the contractions duration is ?
B . Adenomyosis . A . 22 _ 35 sec
C . Endometrial polyps . B . 35 _ 45 sec
D . Cytocele . C . 45 _ 65 sec
‫ اﺳﺎﻣﻪ اﻟﺨﺰاﻋﻠﻪ‬: ‫اﻟﺪﻛﺘﻮر‬ ‫اﺳﺌﻠﺔ اﻟﻨﺴﺎﺋﻴﺔ واﻟﺘﻮﻟﻴﺪ‬
Page 22
D . 56 _ 68 sec E . Krukenberg tumor
E . 85 _ 95 sec
* The commonest cause of protraction disorders
* All of the followings are risk factor for during labor is ?
epithelial ovarian cancer except ? A . Postterm pregnancy .
A . Family history of breast cancer . B . Inadequate uterine activity .
B . Prolonged intervals of ovulation not C . Multiparity .
interrupted by pregnancy . D . Large sized fetus .
C . Infertile females . E . Assisted reproductive technique pregnancy
D . Early age of menopause .
E . White race ladies . * Toxoplasmosis is transmitted mostly from ?
A . Cat's
* The major risk of External cephalic version if B . Dogs
done at the proper time is ? C . Birds
A . Preterm labor . D . Cow's
B . Abruptio placentae . E . Monkeys
C . Preterm ROM .
D . Cord compression . * The most common symptom of vulvar cancer
E . Change the type of Breech is
A . Abnormal bleeding .
* Oligohydramnios can be found in all of the B . Itching .
following except ? C . Palpable lesion on the vulva .
A . Fetal renal agenesis . D . Dyspareunia .
B . Postdate pregnancy . E . Visible abnormality of the vulva .
C . Rupture of membranes .
D . Anencephaly . * The most degenerative changes that occur to
E . Fetal posterior urethral valve an intramural fibroid in pregnant lady is ?
A . Hyaline acellularity .
* After multifetal pregnancy has been first B . Red degeneration .
diagnosed early in gestation , the most important C . Calcification .
next step is ? D . Fatty degeneration .
A . Determination of Chorionicity . E . Cystic degenerative .
B . Check the gender of the twin .
C . Diagnosis of possible anomalies . * Among the followings list of autoimmune
D . Localization of the placenta . diseases in pregnancy , one of these disease has
E . Rule out Twin Twin Transfusion Syndrome no antibodies that cross the placenta ?
A . SLE .
* A 32 year old , gravida 3 , para 2 , presented at B . ITP .
36 weeks gestation vaginal bleeding . On C . Myasthenia gravis .
examination the uterus is rigid and the fetus is D . Graves disease .
dead . Pelvic examination reveals the cervix to be E . Rheumatoid arthritis .
effaced and 4 cm dilated . The next appropriate
action is ? * When a 45 years old lady , gravida 6 , para 6 ,
A . Artificial rupture of membranes . presented with typically complaint of severe
B . Proceed immediately to cesarean section . secondary dysmenorrhea and heavy menstrual
C . Start Clexane therapy to break the cycle of bleeding together with Dyspareunia . The logic
DIC . first diagnosis must be jump in mind will be ?
D . Stabilization , full workup , and manage A . Adenomyosis .
accordingly . B . Endometrial cancer .
E . Induce labor by prostaglandin suppositories C . Endometriosis .
then syntocinon drip D . Uterine fibroids .
E . Ectopic pregnancy .
* Signet ring cell are seen in the cancer ovarian
stroma of ? * The most common massive prenatal
A . Dysgerminoma diagnostic procedure performed during
B . Immature teratoma pregnancy is ?
C . Endometroid sinus tumor A . Chorionic villus sampling .
D . Brenner tumors B . Percutaneous umbilical blood sampling .
‫ اﺳﺎﻣﻪ اﻟﺨﺰاﻋﻠﻪ‬: ‫اﻟﺪﻛﺘﻮر‬ ‫اﺳﺌﻠﺔ اﻟﻨﺴﺎﺋﻴﺔ واﻟﺘﻮﻟﻴﺪ‬
Page 23
C . Doppler sonography . C . Diabetes insipidus .
D . Three dimensional Ultrasonography . D . Severe liver cirrhosis .
E . Amniocentesis . E . Migraine headache with focal neurological
symptoms .
* In reviewing the characters of normal labor
there will be no difference in one of the following * All of the following are normal physiological
parameters between primipara and multipara ? changes in the respiratory system in pregnancy
A . The time of cervical effacement . except ?
B . Duration of the first stage . A . Increase inspiratory capacity .
C . Duration of second stage . B . Decrease functional residual capacity .
D . Duration of third stage . C . Decrease minute ventilation .
E . Rate of cervical dilation during active phase . D . Almost no changes in the vital capacity .
E . Decrease residual volume .
* Among the invasive obstetric procedure
performed during the antenatal period of * When 5 cm , simple cystic adnexal mass is
pregnancy , the ideal time to perform Chorionic noted by Ultrasonography regresses over the
villus sampling is ? course of next cycles . The most likely diagnosis
A . Between 8 to 10 weeks of gestation . is ?
B . Between 11 to 14 weeks of gestation . A . Mucinous cyst .
C . Between 14 to 16 weeks of gestation . B . Dermoid cyst mature type .
D . At 16 _ 20 weeks of gestation . C . Ovarian fibroma .
E . After 20 weeks of gestation . D . Endometrioma .
E . Functional ovarian cyst .
* Each female reproductive cycle ( menstrual
cycle ) represent a complex interaction between * When a lady presented with malodorous non
the following structures except ? irritating discharge and examination reveal
A . Hypothalamus . homogeneous gray white secretion . Mostly this
B . Pituitary gland . lady having ?
C . Ovaries . A . Gardnerella vaginalis infection .
D . Endometrium . B . Vulvovaginal candidiasis .
E . Thyroid gland . C . Trichomonas .
D . Chlamydia trachomatis .
* Each female reproductive cycle ( menstrual E . Neisseria gonorrhoeae infection .
cycle ) represent a complex interaction between
the following structures except ? * The female fetus has the highest lifetime
A . Hypothalamus . number of oocyte at ?
B . Pituitary gland . A . At 20 weeks of gestation .
C . Ovaries . B . Fertilization / Zygote formation .
D . Endometrium . C . Time delivery .
E . Thyroid gland . D . Age of menarche .
E . Reproductive years .
* Amenorrhea in a 28 year old with a high LH and
high FSH is a most recognized feature of ? * Regarding primary dysmenorrhea , one is true
A . Asherman syndrome . A . It usually seen in the third and fourth decay of
B . Sheehan syndrome . females life .
C . Hypothyroidism . B . It occurs few days before the cycle and is
D . Ovarian failure . relieved by the onset of the cycle .
E . Hyperprolactinemia . C . It is dull aching pain in nature .
D . Can be relieved by contraceptive pills .
E . It is relieved after marriage .

* All of the following are absolute


contraindications for COCP except ? * Menorrhagia means ?
A . History of cerebrovascular accident . A . Heavy bleeding .
B . Atrial fibrillation . B . Abnormal vaginal bleeding .
‫ اﺳﺎﻣﻪ اﻟﺨﺰاﻋﻠﻪ‬: ‫اﻟﺪﻛﺘﻮر‬ ‫اﺳﺌﻠﺔ اﻟﻨﺴﺎﺋﻴﺔ واﻟﺘﻮﻟﻴﺪ‬
Page 24
C . Vaginal bleeding with clot . B . Somnolence .
D . Heavy regular vaginal bleeding . C . Slurred speech .
E . Infrequent heavy bleeding . D . Loss of patellar reflex .
E . Paralysis and respiratory difficulty .
* The most common cause of postmenopausal
bleeding is ? * Regarding eclampsia , only one of the following
A . Exogenous estrogens . is correct ?
B . Endometrial cancer . A . Always occur postpartum .
C . Endometrial hyperplasia . B . Urinary output is increase .
D . Endometrial or cervical polyps . C . Antidiuretic drugs are essential in all case's .
E . Atrophic vaginitis . D . Cesarean section is treatment of choice .
E.Magnesium sulfate should be used postpartum
* The leading diameter in occiputo_anterior
presentation is ? * A 27 years old lady G3P1 at 35 weeks
A . Submento vertical . gestation come to emergency room with mild
B . Suboccipitobregmatic . painless vaginal bleeding , BP 110/60 with 1
C . Occiputofrontal . urine dipstick . The most likely diagnosis is ?
D . Biparietal diameter . A . PROM .
E . Submentobregmatic . B . Placenta previa .
C . Abruptio placentae .
* Laparoscopy is an important tool in the D . Polyhydramnios .
diagnosis / management of all of the following E . Preterm labor .
conditions except ?
A . Tubal sterilization . * All of the following indication of delivery in
B . Ectopic pregnancy . preterm premature rupture of membranes except
C . Endometriosis . A . Meconium liquor .
D . Ovarian neoplasms . B . Vaginal bleeding .
E . Pelvic pain . C . Elevated WBCs with increase neutrophils .
D . Fetal distress .
* Regarding drugs in pregnancy , only one of the E . Oblique lie .
following is safe ?
A . Methotrexate . * Complications of Abruptio placentae include
B . Isotretinoin . all of the following except ?
C . Misoprostol . A . Hemorrhage .
D . Progesterone . B . PET .
E . Ribavirin . C . Oliguria .
D . Placental Coagulopathy .
* One of the following conditions is not E . Fetal death .
associated with the development of endometrial
hyperplasia ? * The initial step in the management of
A .Prolonged use of Combined oral contraceptive overactive bladder starts with ?
pills . A . Anticholinergics drugs .
B . Polycystic ovarian syndrome . B . Antimuscarinics drugs .
C . Obesity . C . Functional electrical stimulation .
D . Tamoxifen . D . Botulinum toxin A injection .
E . Granulosa theca cell tumor . E . Behavior modification .

* Regarding the different pathways that thought * All of the followings will cause fetal
to cause preterm birth , the least important one tachycardia except ?
among of the following is ? A . Maternal sedation .
A . Vitamin B12 deficiency . B . Maternal hyperthyroidism .
B . Infection . C . Maternal fever .
C . Placental _ vascular . D . Maternal anxiety .
D . Psychosocial stress and work strain . E . Fetal hypoxia .
E . Uterine stretch .
* The early manifestation of Magnesium sulfate * All of the following increase risk of postpartum
toxicity during management of preeclampsia ? hemorrhage except ?
A . Warmth and flushing . A . Antepartum hemorrhage .
‫ اﺳﺎﻣﻪ اﻟﺨﺰاﻋﻠﻪ‬: ‫اﻟﺪﻛﺘﻮر‬ ‫اﺳﺌﻠﺔ اﻟﻨﺴﺎﺋﻴﺔ واﻟﺘﻮﻟﻴﺪ‬
Page 25
B . Previous postpartum hemorrhage . decrease ability to control flow this is the best
C . Calcified placenta . description for ????
D . Prolonged labour . A . Overflow urinary incontinence .
E . Multiple gestation . B . Stress urinary incontinence .
C . True urinary incontinence .
* In severe preeclampsia , one is true ? D . Urge urinary incontinence .
A . BP 140/90 with +1 protein in urine . E . Normal habits of urination .
B . BP 150/90 with +2 gram protein in urine .
C . BP 150/90 with +2 protein in urine . * In the interpretation of the CTG during fetal
D . BP 160/110 with headache blurred vision . evaluation , the normal result documented
E . BP 140/95 with trace protein in urine . include all of the following parameters except ?
A . At least 2 fetal movement .
* For universal screening and diagnosis of B . Small deceleration ( less than 20 beats ) could
gestational diabetes mellitus in pregnancy be normal .
among normal population whom have no risk C . There should be no evidence of sinusoidal
factors , the first step involve screening test at ? fetal heart rate rhythm .
A . Gestational age 10 _ 12 weeks . D . Beat _ beat variability less than 5 bpm .
B . Gestational age 8 _ 10 weeks . E . The basal heart rate should be between 110 _
C . Gestational age 14 _ 16 weeks . 160 beats / minute .
D . Gestational age 24 _ 28 weeks .
E . Gestational age 18 _ 22 weeks * A 30 year old lady, G1P0, 7wks pregnant
presented to the emergency room wth persistent
* All of the following benign vulva condition are nausea and vomiting, al of the following could be
known to be a white colored lesion except ? the differential diagnosis for her clinical situation
A . Lichen simplex chronicus . except ?
B . Lichen planus . 1- Gastritis.
C . Lichen sclerosus . 2-Missed abortion.
D . Vitiligo . 3-Emesis gravidarum,
E . Acanthosis nigricans . 4- Hyperthyroidism
5-cholecystitis
* Regarding Antepartum hemorrhage all of the
following are correct except ? * All of the following findings could be part of
A . Is vaginal bleeding after 24 weeks . sever PET except :
B . Is vaginal bleeding before delivery of the fetus 1-Polyhydramnios.
C . Passage of show is possible cause . 2- Persistent headache
D . Associated with increased perinatal morbidity 3-Thrombocytopenia.
E . All of pregnancies should be delivered by 4- Epigastric pain
cesarean section 5-Pulmonary edema.

* All of the following are normal physiological * The most common presenting symptom in
changes in pregnancy except ? cases of vulvar carcinoma is:
A . Almost no significant changes in respiratory 1-Vulvar bleeding
rate . 2-Dysuria
B . Increase in Red blood cell mass . 3-Vulvar discharge
C . Increase in WBCs count . 4-Dysparunia
D . Increase in glomerular filtration rate . 5-pruritus
E . Increase gastric emptying time .
* One is true about the uterus after delivery of
* Anti D immunoglobulin is given to all pregnant the fast baby:
women in the following list who are Rh D - and 1-firm
who are not sensitized ( Anti D - ) except ? 2-Irregular surface
A . Ectopic pregnancy . 3- Not painful.
B . Spontaneous or induced abortion . 4-Deviated to left
C . Complete molar pregnancy . 5-above the umbilicus
D . If patient underwent Amniocentesis .
E . If the lady exposed to abdominal trauma .
* When problems with bladder control * Uterine fibroids can be presented by all of the
associated with a strong desire to urine with folowngs except:
‫ اﺳﺎﻣﻪ اﻟﺨﺰاﻋﻠﻪ‬: ‫اﻟﺪﻛﺘﻮر‬ ‫اﺳﺌﻠﺔ اﻟﻨﺴﺎﺋﻴﺔ واﻟﺘﻮﻟﻴﺪ‬
Page 26
1-Accidental finding on ultrasound 1-Pelvic or abdominal pain
2- They cause hypomenorrhea 2- Abnormal vaginal bleeding.
3- Pelvic pain 3-urinary frequency or urgency.
4- infertility 4- bloating
5-Dysmenorrhea. 5- Increased abdominal contour

* The most important reservoir of infection for * The use of oxytocin for the induction and
most of the diseases of humans is augmentation of labor, could be associated with
1- Birds al of the folowings except:
2- Humans 1- Hyper stimulation,
3-Dogs 2- Fetal distress,
4-Fishes 3- Tetanic contraction.
5- Mosquito 4-Mal position.
5- Uterine muscle fatigue which will lead to
* Brow presentation associated with one of the increase the risk of a postpartum hemorrhage.
one of the following:
1-Previous 2 cesarean section * Which of the folowing statements about
2- Hydrocephaly. polyvalent anti- snake venom serum is true?
3- Intrauterine growth restriction (IUGR) 1- It is to be given subcutaneously
4- polyhydramnios. 2-It is an example of passive immunization
5- Large baby 3- is usually obtained from human donors
4- should be stored at 0
* Pelvic floor support is provided mainly by 5- it gives lifelong immunity
1- uterosacral ligament
2- Round ligament * The most commonly encountered anomaly
3-Levator ani muscle. resulting from abnormalites in the development
4- Cervical ligament or canalization of the Mulerian ducts.
5- The bones of the pelvis. 1- Vaginal septurm.
2- Uni-cornuate uterus.
* Which of the following would suggest placental 3-imperforated hymen
abruption? 4-Hermaphroditism.
1-Mal lie 5-vaginal agenesis.
2-Abdominal Tenderness
3- Post date * As normal pregnancy progresses one of the
4-Low placenta folowing hematological changes occur:
5- Painless bleeding 1- Plasma volume increases and red cell volume
remains constant
* Regardind placenta previa, all the following are 2- Plasma volume increase proportionately less
true except: than the increase
1- usually painless recurrent vaginal bleeding 3-red Red cell volume decreases and plasma
2- Usually diagnosed by vaginal examination. volume remains constant
3-May cause maternal death 4- Neither plasma volume nor red cell volume
4- Its incidence is increased with increase panty change
5- The delivery is by cesarean section 5- Plasma volume increases and red blood cell
mass increases
* The optimal frequency of uterine contractions
during the 1st stage of labor is * AI of the folowings could happen with
1- 1-2 contraction/ 10 minutes, injectable progestine contraceptives include al
2- 3-4 contractions/ 10 minutes. the folowing Except:
3- 6-8 contraction / 10 minutes 1- Amenorrhoea.
4- 10-12 contractions/ 10 minutes, 2-Heavier menstrual bleeding during the first few
5- 15-20 contractions/ 10 minutes weeks after device insertion.
3- Headache and nervousness
4- Abdominal bloating.
5-Secondary dysmenorrhae

* AIl the folowings are parameters in Ovarian * When a lady developed post partum
Cancer Symptom Index Except: haemorrhage. al the following measurements
‫ اﺳﺎﻣﻪ اﻟﺨﺰاﻋﻠﻪ‬: ‫اﻟﺪﻛﺘﻮر‬ ‫اﺳﺌﻠﺔ اﻟﻨﺴﺎﺋﻴﺔ واﻟﺘﻮﻟﻴﺪ‬
Page 27
could be part of the management protocol 1-250 mg.
except: 2-80 mg
1-Misoprostol (Cytotic) rectally or intravaginally 3- 1000 mg.
2- Magneesium Sulfate intravenous infusion 4-1500 mg
3- Oxytocin infusion 5-2000 mg.
4- Methergine Methergine
5-uterine massage. * One of these drugs is a teratogenic:
1-Bromocriptine.
* In ovarian hyper-stimulation syndrome al the 2-Methyldopa
following develop except : 3-Metronidazole
1-Large ovarian size 4- Misoprostol.
2-Hydrothorax many develop 5-Ferrous gluconate.
3-Ascites may develop
4-thromboembolism * AlI of the following may cause
5-hyperproteinemia oligohydramnios except:
1-Premature rupture of the amniotic sac
* The tertiary prevention consists of 2- Certain chronic high blood pressure and lupus.
1- crisis preparation 3- Placental insuffiency
2- Elimination of specific disease 4-Fetal polycystic kidney disease
3-Early diagnosis and treatment 5-fetal esophageal atresia
4- Rehabilitation after disability
5-Environmental sanitation * When using magnesium sulfate in eclampsia,
the early manifestation of magnesium sulfate
* The diagnosis of menopause is made: toxicty is:
1-4 months after final menstrual period 1- Warmth and flushing
2- 6 months after final menstrual period 2- Slurred speech
3-8 months after final menstrual period 3- Loss of patellar reflex.
4- 10 months after final menstrual period 4-Somnolence
5- 12 months after final menstrual period 5-Paralysis and respiratory difficulty.

* A pregnant lady at the delvery room, the * The differential diagnoses for first trimester
gestational age s 32 weeks, with mid labour bleeding includes al of the folowing except:
pains, you better give her one of the folowings 1-Threatened miscarriage
1- Clindamycin. 2- Missed miscarriage.
2-Dexamethasone. 3- Incomplete miscarriage.
3-Oxytocin 4-ectopic miscarriages
4-Vit K 5- Cervical incompetence
5- Regular pethidine
* The natural defense mechanism of the lower
* In reviewing the risk factors for endometrial genital tract includes al of the folowing except:
carcinoma, the following list is true except: 1-Bacteriostatic effect of the cervical mucus.
1- increased risk in women who have never 2-the night pH of cervical secretion
married before 3-Apposition of the labia and vaginal walls.
2- Increased risk in women with increasing parity 4- Natural resistance of stratified squamous
3-Increased risk in women with late menopause epithelium
4-Decreased risk with smoking 5- Regular menstruation
5-There is weak evidence of familial tendency
* AI are tests done for Down's syndrome except:
* Investigations for recurrent miscarriages, 1- Maternal serum fetal cells.
include al the following 2- Serum inhibin
1- TORCH screening. 3-Serum B -hCG
2-Thrombophilia screening 4-CA 125.
3-Thyroid antebodies. 5-Serum estriol
4-Karyotyping of both parents Screening
5-screening for anti-phospholipids.

* The total iron requirements during pregnancy


is
‫ اﺳﺎﻣﻪ اﻟﺨﺰاﻋﻠﻪ‬: ‫اﻟﺪﻛﺘﻮر‬ ‫اﺳﺌﻠﺔ اﻟﻨﺴﺎﺋﻴﺔ واﻟﺘﻮﻟﻴﺪ‬
Page 28
* 20 year old single female, with no menstrual * A 28 year old woman, gravida 2 para 1,
cycle at the clinic, when she was examined presented in labor at 38 weeks gestation
clinically, al secondary sexual characteristics developed fetal distress after rupturing the
were present, the next step is membranes, her vaginal examination showed -2
1- Brain computed axial tomography (CAT) station, cephalo presentation 8-9 am dilated
2- progesterone with drawl test cervix. The next appropriate step is
3-Intravenous pyelogram (vP) 1-Apply fundal presuure,
4- Tomography of the adrenal gland 2- Application of forceps
5-Karyotyping 3- Application of vacuum.
4-Immediate cesarean section.
* All of the followings are signs of severe 5- oxytocin augmentation of uterine activity, and
preeclampsia toxemia except: encouragement of pushing
1- Blood pressure of 140/90 mmHg,
2- sever right upper quadrant pain * The dominator in breech presentation is:
3- Blurring of vision. 1- Symphysis pubis
4-Thrombocytopenia 2-Sacrum
5- severe headache. 3-Coccyx
4-iliac crest
* AI of the followings are physiological changes 5- Buttocks.
of pregnancy
1-Increases in the cardiac output * One of the folowings doesn't norease fungal
2- increase in the glomerular filtration rate. vaginal infestation
3- decrease blood pressure in the 3 trimester 1- diabetes mellitus
4-Increase white blood cells count 2-Intrauterine contraceptave device
5- Delayed gastric emptying 3- Broad spectrum antibiotics.
4-Corticosteroids
* The accuracy for dating a pregnancy in the 5- oral contraceptive pills.
second trimester
1- Plus, minus 2-3 days. * The universal soreening and diagnosis of
2- Plus, minus 3-5 days. Gestational Diabetes Mellitus in pregnancy, is at:
3- Plus minus 5-7 days 1- Gestational age 10-12 weeks
4- Plus, minus 7-10 days 2- Gestational age 14-16 weeks
5-Plus minus 14-21 days 3-Gestational age 20- 24 weeks.
4-Gestational age 24-28 weeks
* The commonest cause of detrusor over activity 5-Gestational age 18- 22 weeks.
is:
1-Low estrogen level. * Concerning Cystocele, al are true except
2-incontinence surgery. 1-is a prolapse of the bladder in upper part of
3-Upper motor neuron lesion anterior vaginal wall
4- Idiopathic 2-is common after the menopause
5-Recurrent infections 3- Is the cause of stress incontinence of urine
4- May lead to urinary infection
* Al the folowing are risk factors for primary 5-ls not common in nulliparous women
postpartum hemorrhage Except:
1-Prolonged labor * AlI of the followings can cause polyhydramnios
2- History of a previous postpartum hemorrhage except:
3- Fibroid uterus 1-Diabetes with pregnancy
4- Premature labor. 2-Gestational diabetes.
5- Placental abruption. 3- Idiopathic. a
4-Esophageal atresia.
5-Intrauterine growth restriction

We say the fetus has bradycardia, when the base


Iine is below
1-110 bpm.
2-130 bpm
3- 150 bpm.
4- 170 bpm.
‫ اﺳﺎﻣﻪ اﻟﺨﺰاﻋﻠﻪ‬: ‫اﻟﺪﻛﺘﻮر‬ ‫اﺳﺌﻠﺔ اﻟﻨﺴﺎﺋﻴﺔ واﻟﺘﻮﻟﻴﺪ‬
Page 29
5- 190 bpm, 4- 7/2/2018
* We say the fetus has bradycardia, when the 5- 10/5/2017
base Iine is below * The major cause of unplanned pregnancies in
1-110 bpm. women with oral contraception is ?
2-130 bpm A . Breakthrough ovulation at midcycles .
3- 150 bpm. B . High frequency of intercourse .
4- 170 bpm. C . Incorrect use of oral contraception .
5- 190 bpm, D . Malabsorption
E . Vomiting .
* A 30 year old lady GSP4 , al were by cesarean
delivery, she came to the emergency room with * The ranking cause of death in postmenopausal
abdominal pain at 36 weeks gestation. Al of the women is ?
following can the cause except: A . Coronary heart disease
1-Abruption B . Endometrial cancer
2-Uterine wall rupture. C . Carcinoma of breast
3- Uterine wall dehiscence, D . Hip fracture
4- labour pains E . Stroke
5- Fetal distress
* Risk factors for fetal Macrosomia include all of
* The menstrual cycle represents is an the following except ?
interaction between the following structures A . Gestational age
except: B . Maternal smoking
1- Hypo-thalamus. C . Excessive maternal weight gain
2-Pituitary gland a D . Multiparity
3-Thyroid gland. E . Macrosomia in a previous infant
4-Ovaries.
5- Endometrium * All of the following are contraindicated in
pregnancy except ?
* Which of the following is most likely to be the A . Tetracycline
cause of a in a 47 year old woman B . Digoxin
1- Ovarian tumour. C . Warfarin in first trimester
2-Adenomyosis. D . Statin
3-intrauterine contraceptive device (UCD) E . Carbimazole
4- vaginal prolapse.
5- Endometrial polyp. * The main health hazard of menopause is ?
A . Cardiovascular diseases
* All of the followings are contraindications for B . Endometrial cancer
vacuum delivery except C . Hot flushes
1- Pre-terms D . Depression
2- Occipito posterior positions. E . Osteoporosis
3-Brow presentation.
4-Face presentation. * All of the following are absolute
5- Breech presentation, contraindication for HRT use except ?
A . History of venous thrombosis
* All of the following are symptoms of ectopic B . Pregnancy
pregnancy except : C . Undiagnosed dysfunctional uterine bleeding
1-Shoulder tip pain. D . Active liver disease
2-vaginal spotting E . Suspected breast cancer
3- Abdominal pain.
4-Dizziness, * Habitual abortion can result from all the
5- The presence of an Intrauterine contraceptive following except ?
device (UCD). A . Infections
B . Subserous myomas
* If the first day of the last menstrual period of a C . Chromosomal abnormalities
pregnant lady is 2/7/2017. EDD ? D . Hormonal dysfunction
1- 9/7/2017. E . Cervical incompetence
2- 2/4/2017
3- 9/4/2018
‫ اﺳﺎﻣﻪ اﻟﺨﺰاﻋﻠﻪ‬: ‫اﻟﺪﻛﺘﻮر‬ ‫اﺳﺌﻠﺔ اﻟﻨﺴﺎﺋﻴﺔ واﻟﺘﻮﻟﻴﺪ‬
Page 30
* Lower leg edema during the last trimester of * Women with PCOS have a high risk of which
pregnancy is mainly due to ? one of the following ?
A . Complete venous obstruction A . Sexual dysfunction
B . Lymphedema B . Pituitary adenoma
C . Intra_abdominal tumor C . Adrenal hyperplasia
D . Increased venous pressure due to partial D . Impaired glucose tolerance
obstruction E . Ovarian malignancy
E . Arterial obstruction in pelvis
* All of the following are true for Atrophic
* All of the following signs and symptoms in vaginitis except ?
pregnancy should be reported immediately as a A . It is usually caused by estrogen deficiency .
potential danger signal except ? B . It is often asymptomatic .
A . Swelling of ankles and feet C . Spotting or bleeding exclude Atrophic vaginitis
B . Vaginal bleeding as a cause .
C . Severe headache D . Oral estrogen replacement is used to treat
D . Blurring of vision severe symptoms
E . Escape of fluid from vagina E . It can present with dysparunia

* All of the following are true for osteoporosis * All of the following are true for breast pain
except ? except ?
A . Osteoporosis is an abnormal bone A . Breast pain is common and may interfere with
mineralization . quality of life .
B . Osteoporosis is usually asymptomatic . B . Breast cancer is unlikely to present with pain
C . Can result from long term corticosteroids use without associated mass .
D . It is more frequent in thin people . C . Breast feeding should be discontinue on the
E . Serum alkaline phosphatase is usually normal affected breast if the cause of pain is mastitis .
D . More than half of women with premenstrual
* Risk factors for breast cancer include all of the syndrome report breast pain as
following except ? predominant symptom .
A . Breast feeding E . Spontaneous resolution of cyclical breast pain
B . Early menarche occurs in majority of cases .
C . First pregnancy after 30 years age
D . Late menopause * All of the following are true for ectopic
E . Nulliparity pregnancy except ?
A . The fertilized ovum implants in the fallopian
* The following factors would put a 30 weeks tubes in 50 % of cases .
gestation pregnant lady into a high risk category B . Presence of intra_uterine device is a risk
except ? factor .
A . Anemia C . Transvaginal ultrasound is the initial
B . Breech presentation diagnostic test .
C . Weight gain of 0.1 kg per week D . The onest of pain is usually at 6 to 8th weeks
D . Glycosuria on 3 occasions gestational age .
E . Proteinuria on 3 occasions in the absence of E . Acute appendicitis is one of differential
HTN or UTI . diagnosis .

* Contraindication to the use of Combined OCP * All of the following are true for HTN in
include all of the following except ? pregnancy except ?
A . Active liver disease A . It is defined as blood pressure > 140/90
B . SLE measured on more than one occasion .
C . Age above 35 and smokers B . HTN occurring before 20 weeks gestation is
D . Porphyria classified as chronic HTN .
E . PCOS C . Pregnant women should be screened for HTN
every visit .
D . HTN with an onest after 20 weeks gestation is
always classified as preeclampsia .
E . Dihydropyridine CCB are safe in pregnancy .
‫ اﺳﺎﻣﻪ اﻟﺨﺰاﻋﻠﻪ‬: ‫اﻟﺪﻛﺘﻮر‬ ‫اﺳﺌﻠﺔ اﻟﻨﺴﺎﺋﻴﺔ واﻟﺘﻮﻟﻴﺪ‬
Page 31
* All of the following are true for asymptomatic
bacteruria except ? * Most common indication for primary cesarean
A . It occurs in about one third of pregnant section is ?
women . A . Cord prolapse .
B . A urine dipstick is an adequate screening test B . Diabetes .
C . The most causative agent is E coli . C . Previous scar .
D . Nitrofurantoin is effective treatment . D . Dystocia .
E . Treatment reduces incidence of preterm E . Malpresentation .

* All of the following are true for fibroids * Which of the following is associated with small
except ? infants ?
A . Can present with Menorrhagia . A . Mother with untreated gestational Dm .
B . Can be the underlying cause for infertility . B . Multiparity .
C . Increase risk of miscarriage C . Large parents .
D . Obesity is a risk factor D . Maternal smoking .
E . May presents with difficulty in passing urine E . Postdate pregnancy .

* All of the following are true for chlamydia * Most common presentation in case of Abruptio
infection except ? placentae ?
A . Majority of cases are asymptomatic . A . Breech .
B . May present with postcoital bleeding . B . Cephalic .
C . It is a preventable cause of infertility . C . Shoulder .
D . Urine sample can be used for screening D . Arm .
asymptomatic women . E . Mixed .
E . Mucopurulent cervicitis is found on
examination * A 23 year old patient , amenorrheic for 16
weeks , had vaginal spotting . She was found to
* All of the following are true for smoking in have a uterus rise into the umbilicus and fetal
pregnancy except ? heart tone audible by Fetoscope . HCG serum
A . It increases risk of miscarriage levels were approximately 200 IU . Which of the
B . It increases risk of ectopic pregnancy following tests is the most appropriate ?
C . It decreases birth weight A . HCS .
D . It increases risk of perinatal mortality B. Pelvic ultrasound .
E . Smoking has no relation with placenta previa C . Serial HCG .
D . Serial clotting function studies .
* A 30 year old Methamphetamine user presents E . APT test on vaginal blood
with active labor . She has had no prenatal care .
You check fetal position and feel nose , eyes , * What is the maximum normal time for the
mouth . Most common associated condition with second stage of labor in a primigravida with
this presentation is ? epidural anesthesia ?
A . Anencephaly . A . 60 minutes .
B . Hydrocephaly . B . 120 minutes .
C . Prematurity . C . 150 minutes .
D . Placenta previa . D . 180 minutes .
E . Oligohydramnios . E . 240 minutes .

* All of the following are true about Cytocele * The M.C encountered anomaly resulting from
except ? abnormalities in the development or canalization
A . Associated with weakness in the broad of the paramesonephric duct is ?
ligament . A . Vaginal septum .
B . Protrusion of bladder into the anterior vaginal B . Unicornuate uterus .
wall . C . Imperforated hymen .
C . Associated with frequency , uregency , D . Hermaphroditism .
nocturia . E . Vaginal agenesis .
D . Common risk factor is multiple vaginal .
E . First step in treatment is Kegel exercises , and * All of the following are teratogenic drug except
if failed so anterior Colporrhaphy is the best A . Thalidomide .
‫ اﺳﺎﻣﻪ اﻟﺨﺰاﻋﻠﻪ‬: ‫اﻟﺪﻛﺘﻮر‬ ‫اﺳﺌﻠﺔ اﻟﻨﺴﺎﺋﻴﺔ واﻟﺘﻮﻟﻴﺪ‬
Page 32
B . Misoprostol .
C . Carbimazole .
D . Fluconazole .
* The muscle that provide the best support to * During the menstrual cycle , the histological
pelvic floor is ? appearance of the endometrium will change
A . Levator ani . significantly , During the first half of the cycle .
B . Rectovaginal . The endometrium becomes thicker and rebuilds
C . Coccygeus . largely in response to ?
D . Piriformis . A . Progesterone .
E . Obturator internus . B . Estrogen .
C . Luteinizing hormone .
* Most common reservoir of infection for human D . Gonadotropin releasing hormone .
A . Mosquito . E . Follicle stimulating hormone .
B . Cat's .
C . Dog's . * Implantation in the uterus occurs at which
D . Bacteria . stage of development ?
E . Human . A . Eight cell embryo .
B . Zygote .
* During an evaluation for infertility , a woman C . Morula .
may have an endometrial biopsy to evaluate the D . Blastocyst .
quality of her ovulation . The optimal E . Embryonic disc .
development of the corpus luteum is most
closely associated with ? * During pregnancy , what is the safest analgesic
A . Fertilization of an ovum . with the least fetal effect ?
B . Follicular phase of the endometrium . A . Aspirin .
C . Luteal phase of the endometrium . B . Acetaminophen .
D . Secretory phase of the endometrium . C . Ibuprofen .
E . Shedding phase of the endometrium D . Gabapentin .
E . Diclofenac sodium .
* In the fetus, the most well oxygenated blood is
allowed into the systemic circulation by ? * A 28 year old woman , gravida 2 para 1 .
A. Ductus arteriosus. Presented in labor at 38 weeks gestation
B. Foramen ovale. developed fetal distress after rupturing the
C. Rt. Ventricle. membranes , her vaginal exam showed _ 2
D. Ligamentum teres. station , Cephalic presentation , dilated cervix ,
E. Ligamentum venosum the next appropriate step is ?
A . Apply fundal pressure .
* All are associations with Polyhydramnios B . Application of forceps .
except ? C . Application of vacuum .
A . Tracheosesophageal fistula . D . Immediate cesarean section .
B . Anencephaly . E . Oxytocin augmentation of uterine activity and
C . TTT syndrome . encouragement of pushing .
D . IUGR .
E . Rh isoimmunization . * Denominator in vertex , breech , and face
presentation respectively are ?
* All of the following are contraindications for A . Occiput , Sacrum , mentum .
tocolysis in preterm labor except ? B . Occiput , mentum , Sacrum .
A . Severe preeclampsia . C . Mentum , Sacrum , occiput .
B . Chorioamnionitis . D . Mentum , occiput , Sacrum .
C . Cervical dilation 1 cm . E . Sacrum , mentum , occiput .
D . Severe IUGR .
E . IUFD . * In the normal labor , the pressure produced by
uterine contractions is greatest at which of the
following time's ?
A . Latent phase .
B . Active phase .
C . Second stage .
D . Third stage .
‫ اﺳﺎﻣﻪ اﻟﺨﺰاﻋﻠﻪ‬: ‫اﻟﺪﻛﺘﻮر‬ ‫اﺳﺌﻠﺔ اﻟﻨﺴﺎﺋﻴﺔ واﻟﺘﻮﻟﻴﺪ‬
Page 33
E . Braxton Hicks sign . B . Dysplasia .
C . Metaplasia .
D . Neoplasia .
* Vaginal examination is contraindicated in E . Vaginal adenosis .
which situation during pregnancy ? * During a pelvic exam , on a patient that is
A . Carcinoma of the cervix . approximately 8 weeks gestation by dates and
B . Prolapsed cord . pelvic examination . One adnexa is found to be
C . Gonorrhea . slightly enlarged . Most likely diagnosis ?
D . Placenta previa . A . Corpus luteum cyst .
E . Abruptio placentae B . Ectopic pregnancy .
C . Follicular cyst .
* A 26 years old woman whose last menstrual D . Ovarian neoplasm .
period was 8 weeks ago develops bleeding , E . Chocolate cyst .
abdominal cramps , and passes tissue per vagina
two hours later , she is still bleeding , bleeding * A 21 year old girl is seen for her first prenatal
heavily , the most likely diagnosis is ? visit at 34 weeks gestation . On exam her BP is
A . Twin pregnancy . 148 / 96 mmhg and her fundus measures 31
B . Threatened abortion . cm . Her urine dipstick is +3 for protein . Which of
C . Inevitable abortion . the following is the most likely diagnosis ?
D . Premature labor . A . Hypertensive disease with superimposed
E . Incomplete abortion . preeclampsia .
B . Mild eclampsia .
* There is good evidence that a woman who C . Preeclampsia .
gave birth to an infant with a neural tube defect D . Chronic HTN .
can reduce the risk of recurrence by taking E . Third trimester normal pregnancy .
periconceptional folic acid supplementation,
what is the recommended dose ? * During pregnancy blood test for diabetes are
A . 0.4 mg more abnormal than in nonpregnant . Also non
B . 0.8 mg diabetic women may be develop gestational
C . 1 mg diabetes during the last half of the pregnancy .
D . 4 mg This is because ?
E . 40 mg A . Decreased insulin production .
B . Increased food absorption from GI .
* Your patient has microcytic anemia with a C . Increased human placental lactogen .
hemoglobin of 9 g/dL and normal iron stores . D . Hemoconcentration .
What is the most likely diagnosis ? E . Decreased hepatic secretion of insulin binding
A . Folate deficiency . globulin .
B . Vitamin B12 deficiency .
C . Thalassemia . * A 34 year old woman at 38 weeks whose
D . Vitamin B6 deficiency . pregnancy is complicated by gestational
E . Acute blood loss . diabetes . She is in labor , the head delivers but
the shoulders do not follow . An effective method
* A patient presents to you , she had a positive of delivery for shoulder dystocia includes
pregnancy test and wonders if you can tell her McRoberts manoeuvre . McRoberts manoeuvre
when she is likely due . The duration of cycle 35 , is described as which of the following ?
the LMP was June 30 . Her expected date of A . Fundal pressure .
delivery is ? B . Extreme flexion of the maternal thighs .
A . March 23 . C . Rotation to an oblique position after delivery
B . April 7 . of posterior arm .
C . April 14 . D . Strong traction on the head .
D . March 14 . E . Rotation of the posterior shoulder to the
E . March 7 . anterior .

* A 35 year old G5 P4 pregnant is found to have


finding on her pap smear . Which of the following
cervical changes . May be found more frequently
in the pregnant than in the nonpregnant ?
A . Atypical glandular hyperplasia .
‫ اﺳﺎﻣﻪ اﻟﺨﺰاﻋﻠﻪ‬: ‫اﻟﺪﻛﺘﻮر‬ ‫اﺳﺌﻠﺔ اﻟﻨﺴﺎﺋﻴﺔ واﻟﺘﻮﻟﻴﺪ‬
Page 34
C . Directed at preventing disability in people who
have symptomatic disease , aiming to improve
quality of life .
D . Aiming at reducing prevalence of non
communicable disease .
* A previously healthy woman complains of reducing complications of non communicable
crying , loss of appetite , difficulty in sleeping , disease .
feeling of low self worth , starting 3 day after a * most common mode of delivery in case pf
normal vaginal delivery and persisted for 2 weeks placenta previa centralis ?
then progressively diminished , most likely A.normal vaginal delivery .
diagnosis is ? B. assisted vaginal delivery .
A . Blues . C. LUS cesarean delivery .
B . Manic depression . D. vertical cesarean delivery .
C . Neurosis . E. cesarean hysterectomy .
D . Psychosis .
E . Schizophrenia . * psammoma bodies in histopathology
indicate ?
* The shortest line from the inside of the A. endometrioid epithelial ovarian cancer .
symphysis to the most prominent point on the B. serous cysadenoma of the ovaries .
front two segment of the sacrum is called ? C. ovarian dysgerminoma .
A . True conjugate . D. choriocarcinoma .
B . Obstetric conjugate . E. thecoma
C . Diagonal conjugate .
D . Bi_ischial diameter . * The optimal time for screening of gestational
E . Oblique diameter . diabetes at ?
A . 20 _ 22 weeks gestation .
* Which artery provides the blood supply to the B . 22 _ 24 weeks gestation .
vulva ? C. 24 _ 28 weeks gestation .
A . Inferior hemorrhoidal artery . D . 28 _ 30 weeks gestation .
B . Ilioinguinal artery . E . 16 _ 20 weeks gestation .
C . Pudendal artery .
D . Femoral artery . * The normal number of uterine contractions in
E . Inferior hypogastric artery . 10 minutes , duration and intensity during second
stage of labor ?
* Near the external os of the cervix , what is A . 2_3 / 10 min , 45 _ 60 sec , 75 mmhg .
found as a normal transition B . 3_5 / 10 min , 75 _ 90 sec , 100 mmhg .
from columnar epithelium ? C . 5_7 /10 min , 45 _ 60 sec , 50 mmhg .
A . Keratinized epithelium . D . 1_3 /10 min , 75 _100 sec , 25 mmhg .
B . Squamous epithelium . E . 2 _ 3 /10 min , 75 _ 100 sec , 50 mmhg .
C . Transitional epithelium .
D . Cuboidal epithelium . * All of the following are risk factors of
E . Cervical erosion . premature rupture of membranes except ?
A . Bacterial vaginosis .
* The area of the skull between the anterior and B . Smoking .
posterior fontanelles , and the parietal eminence C . Multiple gestation .
top of the skull is called ? D . Oligohydramnios .
A . Sagittal suture . E . Amniocentesis .
B . Lambdoid suture .
C . Coronal suture . * Classical clinical presentation of ectopic
D . Vault suture . pregnancy include all of the following except ?
E . Mastoid fontanelle . A . Positive pregnancy test .
B . Amenorrhea 10 _ 12 weeks .
* Tertiary prevention is ? C . Intermittent scanty vaginal bleeding .
A . Aiming at reducing incidence of non D . Abdominal pain or shoulder pain .
communicable disease . E . Hemoperitoneum .
B . Examples are screening programs of cancer ,
and dietary recommendation . * Common side effects of progesterone
intrauterine device are all of the following except
‫ اﺳﺎﻣﻪ اﻟﺨﺰاﻋﻠﻪ‬: ‫اﻟﺪﻛﺘﻮر‬ ‫اﺳﺌﻠﺔ اﻟﻨﺴﺎﺋﻴﺔ واﻟﺘﻮﻟﻴﺪ‬
Page 35
A . Heavy menses . 5- air trael is not allowes for patient triplrt at 34
B . Missed periods . week g gestation
C . Weight gain .
D . Depression .
E . Acne .

* Criteria of preterm labor include all of the * in the management of patient suspected as
following except ? acute pelvic inflammatory disease , all
A . Onest before 37 weeks gestation . investigation are beneficial except
B . Irregular uterine contractions . 1- urine analysis and culture
C . At least 2 _ 3 contractions in 10 minutes . 2-CRP
D . Progressive cervical effacement > 80% . 3-laproscopy
E . Cervical dilation > 1 cm . 4-endocervical swab
5-hysteroscopy
* The indications for vacuum extraction are all of
the following except ? * regarding the bacterial vaginosis all true except
A . Engaged head and no Cephalopelvic 1-commonly presented as deep dysparounia
disproportion . 2-wet smear thows clue cell
B . Face presentation . 3-no need to treat the partner
C . Known fetal head position . 4-is usually asymptomatic
D . Cervix 10 cm dilated . 5-culture commonly reveals gardnerella
E . Membranes rupture in term pregnancy vaginosis

* A patient presents for her routine prenatal visit * during which the stage of labor is the fetus
at 32 weeks EGA . Her laboratory testing is delivered
normal including 1 hour OGTT . Her symphysis 1- stage ZERO
Fundal height is 37 cm today . A bedside 2- stage one
ultrasound reveals an AFI 30 cm . Which of the 3- stage two
following is the most likely etiology ? 4- stage three
A . Renal atresia . 5- stage four
B . Pulmonary hypoplasia .
C . Gestational diabetes . * breast milk containe all except
D . Duodenal atresia . 1-vitm A
E . Urethral stenosis . 2-vtm B
3-vitmD
* A 49 year old parous woman comes 4-vitm e
complaining that over the last several years , it 5-vitm K
feels her organs are progressively falling out her
vagina , she complains of losing urine with * one of the following is NOT a complication of
coughing , urgency . Most likely diagnosis is ? preeclampsia
A . Stress incontinence . 1-HELLP syndrome
B . Urge incontinence . 2-placenta previa
C . Overflow incontinence . 3-pulmonary edema
D . Functional incontinence . 4-acute pulmonary edema
E . Mixed incontinence 5-cerebral hemorrhage

* regarding antenatal care (ANC) the all true * regarding progesterone only pills ,all true
except except
1-regular ANC can prevent ..... macrosomia 1-safe in women over 40
2-regular ANC reduce the incidince of 2- associated with breakthrough bleeding
pylonephtitis 3-should be taken dally at fixed time
3-exercise during pregnancy can help in reduce 4-associated with functional ovarian cyst
weight after delivary 5-as effect as combined pills in inhibition of
4-regular ANC can prevent hydrops fetalis rhesus ovulation
isommunized mother
‫ اﺳﺎﻣﻪ اﻟﺨﺰاﻋﻠﻪ‬: ‫اﻟﺪﻛﺘﻮر‬ ‫اﺳﺌﻠﺔ اﻟﻨﺴﺎﺋﻴﺔ واﻟﺘﻮﻟﻴﺪ‬
Page 36
* low level of HCG may be found in which of the
* which the following antibiotic is following
contraindication during pregnancy 1- ectopic pregnancy
1-amoxicillin 2- down syndrome
2-imipenem 3-erythro blastosis fetalls
3-oxacillin 4-molar pregnancy
4-levofloxacin 5-twin pregnancy
5-ampicilin

* regarding the management labor in women * the heart rate of fetus during labor is
with heart disease , all are correct except : considered to be normal if it
1- ensure adequant analgesia during 1st stage 1- 100-120 bpm
2-fluid restiction is recommended 2- 80-100 bpm
3- ergometrine can be used in the active 3-160-180 bpm
manegment of 3rd stage 4-120-160 bpm
4- keep 2nd stage short
* A 55 y old asymptomatic obese female present
* regarding hydatiform mole all true except : to your clinic for checkup and general advice .ypu
1-is an abnormal growth of the placenta advice her that all of the following condition are
2- in a complete molar pregnancy ,some fetus strongly associated with obesity except :-
development may occurs 1- osteroporosis
3- is due to problem during fertillization 2-atherosclerosis
4-uterus is large in about half of cases 3-hypoventilation syndrome
5- nausea and vomiting is more with complete 4-insulin resistance
mole 5-hypertention

* m c symptom of cerviacl ca : * cushing syndrome is charactriized by all except


1- increase freguency if urination 1-poor wound healing
2- pelvic pain 2-Acne
3- postcoital bleeding 3-hyperpigmentation of the buccal mucosa
4- infertility 4-Hirsutism
5-truncal obesity
* A 26 y old primigravida was found to have 6*6
cm right ovarian cyst during * CO2 mostly carried by :-
routine visit ,at 10 week gestation ,the best 1-HCO3
course of action is 2-albumin
1-admit for ovarian cystectomy immediately 3-globulin
2-rescan at 14-18 week 4-hemoglobin
3-reassure the patient and ignore the cyst as it 5-dissolved in the blood
will disappeare spontaneously
4-give progesterone therapy * at what gestational age dose aminiotic fluid
5-arrange for CT scan and tumor marker vomume reach its peak
1- 24 week
* which the earlist point in pregnancy that a 2- 28 week
woman will begin to perceive fetal movement ? 3- 30 week
1- 8 week 4-34 week
2- 10 week 4- 40 week
3- 16 week
4- 24 week * what is the approximate riske of trisomy 21 in
5- 30 week 20 y old women
1- 1 in 1200
2- 1 in 1000
* low birth weight is : 3- 1 in 500
1- less than 2.00 kg 4- 1 in 150
2- less 2.5 kg and the height less than 40 cm 5- 1 in 70
3- less than 2.5 kg
4- between 1.5-2.5 kg * all the following are STDs (sexually
transmotted disease ) except :
‫ اﺳﺎﻣﻪ اﻟﺨﺰاﻋﻠﻪ‬: ‫اﻟﺪﻛﺘﻮر‬ ‫اﺳﺌﻠﺔ اﻟﻨﺴﺎﺋﻴﺔ واﻟﺘﻮﻟﻴﺪ‬
Page 37
1-trichomonas vaginalis C . Hypothyroidism .
2-syphilis D . Fibroids .
3-chlamydia E . OCPs .
4-candidia albicans
5-gonorrhea * The maximum number of oogonia are formed
at what age female life ?
A . One month intrauterine .
B . Five months intrauterine .
C . At birth .
* 5 y old girl present with foul smelling vaginal D . At puberty .
discharge most likely causes is :- * A primigravida is in second stage of labor for
1-foreign body the past 2 hours . Fetal head is at first station . In
2-candidiasis spite of effective uterine contractions, mother is
3-trichomoniasis unable to push as she is exhausted . The best
4-vesicorectal fistula next step is ?
5-bacterial vaginosis A . Wait for another one hour .
B . Give sedation to the mother .
* ovarian vessels are found in which ligaments :- C . Shift her for emergency cesarean section .
1-broad D . Instrumental delivery .
2-round E . Give oxytocin to the mother .
3-uterosacral
4-infundiabulo pelvic * 23 years old primigravida presents to you at 14
5-broad and round weeks of gestation . She is concerned about
normality of fetus . At what time you will advise
* All of the following are indications for induction her detailed
of labor except ? fetal anomaly scan ?
A . Prolonged pregnancy . A . 22 _ 24 weeks .
B . IUGR . B . 14 _ 16 weeks .
C . Abnormal lie . C . 18 _ 22 weeks .
D . Gestational HTN . D . 10 _ 14 weeks .
E . Term decreased fetal movement . E . 24 _ 28 weeks .

* The best method to diagnose endometriosis is * The most significant risk factor for placental
A . Laparotomy . abruption is ?
B . CT scan . A . Trauma .
C . MRI . B . Polyhydramnios .
D . Ultrasound scan . C . Short cord .
E . Laparoscopy . D . Preeclampsia .
E . Chorioamnionitis .
* The oxygenated blood is allowed into the fetal
body by ? * The most frequent symptom of ectopic
A . Foramen ovale . pregnancy ?
B . Ductus venosus . A . Dizziness .
C . Right ventricle . B . Vaginal bleeding .
D . Umbilical arteries . C . Abdominal pain .
E . Fetal aorta . D . Collapse .
E . Frequency of micturition .
* Apgar score parameters include all of the
following except ? * Fetal hyperinsulinemia causes of increase
A . Respiratory rate . growth of the following tissue except ?
B . Pulse rate . A . Heart
C . Response to stimuli . B . Liver
D . Colour . C . Brain
E . Tone . D . Subcutaneous fat
E . Muscles
* All are causes of regular heavy periods except
A . Endometrial polyp . * Bishop score include all of the following except
B . Endometriosis . A . Cervical position .
‫ اﺳﺎﻣﻪ اﻟﺨﺰاﻋﻠﻪ‬: ‫اﻟﺪﻛﺘﻮر‬ ‫اﺳﺌﻠﺔ اﻟﻨﺴﺎﺋﻴﺔ واﻟﺘﻮﻟﻴﺪ‬
Page 38
B . Cervical consistency . D . Teratoma .
C . Cervical dilation . E . Granulosa cell tumor .
D . Station of the presenting part .
E . The colour of the amniotic fluid . * All are true for the presentation of over active
bladder except ?
A . Increased urinary frequency .
B . Uregency .
C . Urge incontinence .
D . The sudden desire to void at large urine
volume .
* When a 45 years old lady , gravida 6 , para 6 , E . Infection should be ruled out .
presented with typically complaint of heavy * The elevation of basal body temperature just
menstrual bleeding together with dyspareunia . before ovulation is related mainly to ?
The logic first diagnosis must jump ? A . Central effect of progesterone .
A . Ectopic pregnancy . B . The luteinizing hormone surge .
B . Endometriosis . C . Effect of specific sex hormone binding
C . Endometrial cancer . globulin .
D . Uterine fibroids . D . Immunologic reaction effect of the pituitary
E . Adenomyosis . hormones .
E . Transformation of follicular endometrium into
* All of the following may be found in ovarian secretory endometrium .
hyperstimulation syndrome except ?
A . Large ovaries bilateral . * Regarding antenatal care , all of the following
B . Hydrothorax may develop . statements are correct except ?
C . Ascites may develop . A . Regular antenatal care detect hydrops fetalis
D . Hypovolemia develop . by ultrasound .
E . Hyperproteinemia may develop . B . Regular antenatal care can minimize the risk
of postterm pregnancies .
* Investigations for recurrent abortion may C . Pelvic examination is done routinely .
include all of the following except ? D . Mothers who are seronegative for rubella
A . Transvaginal ultrasound . should be immunized after
B . Thrombophilia screening . delivery .
C . TORCH screening . E . Screening for group B streptococcus reduces
D . Parental karotyping . the risk of neonatal sepsis .

* The best independent predictor of induction of * All of the following are prerequisites for the
labor success at the time of initiation is ? application of instrumental delivery except ?
A . Cervical effacement A . The cervix is dilated 8 cm .
B . Cervical position B . Maternal bladder is empty .
C . Cervical dilation C . The fetal head is engaged .
D . Cervical consistency D . The fetal position is known .
E . Station of the head E . The mother has adequate analgesia .

* The first logic diagnostic test for patient with * All of the following are indications for forceps
excessive bleeding per vagina is ? delivery except ?
A . Pregnancy test . A . Prolonged second stage of labor .
B . Pelvic ultrasonography . B . Macrosomia in gestational diabetes mellitus .
C . Endometrial biopsy . C . Fetal distress in second stage of labor .
D . Complete blood count . D . Maternal cardiac disease .
E . Coagulation tests . E . Poor maternal effort .

* A 45 year old lady complaining of * Regarding cervical incompetence , only one of


breathlessness and who was noted to have the following is correct ?
unilateral ovarian mass . The most likely A . Is a common cause of first trimester
diagnosis is ? abortion .
A . Follicular cyst . B . Can only be diagnosed by clinical history .
B . Fibroma . C . Commonly occur following multiple
C . Serous cystadenoma . pregnancy
‫ اﺳﺎﻣﻪ اﻟﺨﺰاﻋﻠﻪ‬: ‫اﻟﺪﻛﺘﻮر‬ ‫اﺳﺌﻠﺔ اﻟﻨﺴﺎﺋﻴﺔ واﻟﺘﻮﻟﻴﺪ‬
Page 39
D . May be congenital . * All of the following increase during normal
E . Requires a cervical cerclage before 10 weeks pregnancy except ?
gestation . A . Cardiac output .
B . White blood cells .
* Concerning genital prolapse , all are true C . Peripheral vascular resistance .
except ? D . Heart rate .
A.Postnatal physiotherapy should be encouraged E . Red cell mass .
B . Loss of estrogen is a risk factor .
C . Multiparity is a risk factor .
D . Surgical repair is one of the modalities of
treatment .
E . Common with repeated operative deliveries .
* Factors that increase the risk of genital * Factors that increase the risk of
prolapse include all of the following except ? thromboembolism in pregnancy include all the
A . Hysterectomy . following except ?
B . Forceps delivery . A . Suppression of lactation with estrogen .
C . Prolonged labor . B . Prolonged hospital stay .
D . Chronic constipation . C . Induction of labor .
E . Repeated cesarean section . D . Cardiac disease .
E . Thrombophilia .
* In Jordan , which of the following is used for
treatment of drinking water ? * The most common cause of secondary
A . Chlorine . postpartum hemorrhage is ?
B . Iodine . A . Uterine fibroids .
C . Fluorine . B . Coagulation defect .
D . Carbon dioxide . C . Trauma to genital tract .
E . Ozone . D . Infection .
E . Abnormal arteriovenous formation .
* Regarding endometrial carcinoma , all of the
following statements are correct except ? * Regarding bacterial vaginosis , all are true
A . Rare occur before the age of 40 . except ?
B . Adenocarcinoma is the commonest type . A . Clue cell's are present .
C . Usually present with postmenopausal B . It is a sexual transmitted disease .
bleeding . C . Can cause preterm labor .
D . The diagnosis may be suspected by abnormal D . It causes homogeneous , gray and thin
pap smear . vaginal discharge .
E . Usually advanced stage at the time of E . Metronidazole used for treatment .
diagnosis
* In severe preeclampsia , one is true ?
* Regarding staging classification of cervical A . Blood pressure 140/90 with +1 protein in
cancer , all are true except ? urine .
A . Stage one confined to cervix . B . Blood pressure 150/90 with 2 grams protein
B . Stage two B obvious parametrial involvement in urine .
C . Stage three extends to pelvic side wall . C . Blood pressure 150/90 with +2 protein in
D . Stage three extends to lower third of vagina or urine .
hydronephrosis . D . Blood pressure 160/110 with headache and
E . Stage four A metastasis in the lung . blurred vision .
E . Blood pressure 140/95 with trace protein in
* A primigravida admitted with rhythmic uterine urine .
contractions every 5 minutes and cervix is 2 cm
dilated and 1 cm long . She is in ? * All of the following are indications of delivery in
A . Latent phase of labor . preterm rupture of
B . Active phase of labor . membranes except ?
C . Second stage of labor . A . Meconium liquor .
D . Not in labor . B . Vaginal bleeding .
E . Late first stage of labor . C . Elevated white blood cells and increase
neutrophils .
D . Fetal distress .
‫ اﺳﺎﻣﻪ اﻟﺨﺰاﻋﻠﻪ‬: ‫اﻟﺪﻛﺘﻮر‬ ‫اﺳﺌﻠﺔ اﻟﻨﺴﺎﺋﻴﺔ واﻟﺘﻮﻟﻴﺪ‬
Page 40
E . Oblique lie . bleeding . The uterus was felt 2 cm below the
umbilicus and firm . The most likely cause is ?
* A 20 years old pregnant lady , G2P1 , lactating A . Uterine atony .
with a missed period of 6 weeks .presented with B . Retained placental tissue .
minimal vaginal bleeding , and abdominal pain . C . DIC .
B_HCG ( 630 ml IU/ml ) , she had stable vital D . Infections .
signs . The best management ? E . Genital tract trauma .
A . Repeat B_HCG after 48 hours .
B . Diagnostic laparoscopy .
C . Laparotomy .
D . Methotrexate .
E . See the mother in one week time .

* In placenta previa , only one statement is * A 39 years old woman para 6 has presented
correct ? with complaint of postcoital bleeding for the past
A . Vaginal examination is safe . three months . Your first investigation should be
B . Abdominal ultrasound is not accurate . A . Dilation and curettage .
C . Uterus is tender on palpation . B . Cone biopsy of cervix .
D . High incidence of intrauterine fetal death . C . Pap smear .
E . High risk of postpartum hemorrhage . D . Colposcopy .
E . Laparoscopy .
* Which artery is considered to be important to
differentiate between site * A 40 years old multiparous woman complains
of origin in inguinal hernia ? of involuntary loss of urine associated with
A . Cremasteric artery . coughing , lifting or standing . The history is most
B . Superior epigastric artery . suggestive of ?
C . Inferior epigastric artery . A . Fistula .
D . Femoral artery . B . Stress incontinence .
E . Superficial iliac circumflex artery . C . Urge incontinence .
D . UTI .
* Principles of the management of septic E . Urethral diverticulum .
abortion include all of the following except ?
A) Cervical swab for culture . * 28 years old woman with previous history of
B ) IV antibiotics . having baby with Down syndrome , she is now 12
C) Correction of hypovolemia . weeks pregnant . Which of the following would
D) Immediate evacuation of the uterus . be suggest ?
E) Cross matching for 2 units of blood . A . Amniocentesis .
B . Obstetric ultrasound .
* Oligohydramnios can be found in all of the C . Chorionic villus sampling .
following except ? D . Fetal blood sampling .
A . Fetal renal agenesis . E . Wait till 18 weeks for detailed ultrasound .
B . Postterm pregnancy .
C . Rupture of membrane .
D . Anencephaly . * Regarding primary dysmenorrhea , one is true
E . Posterior urethral valve anomaly of the fetus . A . It usually seen in the third and fourth decay of
female life .
* After multifetal pregnancy has been first B . It occurs few days before the cycle and is
diagnosed early in gestation . The most relieved by the Onest of the cycle .
important next step is ? C . It is dull aching pain in nature .
A . Determination of chorionicity . D . Can be relieved by contraceptive pills .
B . Check the gender of the twins . E . It is relieved after marriage
C . Diagnosis of possible anomalies .
D . Localization of the placenta . * For universal screening and diagnosis of
E . Rule out TTT syndrome . gestational diabetes mellitus in pregnancy . The
first step involves screening test at ?
* A para 1 , just had vacuum delivery of 3 kg A . Gestational age 10 _ 12 weeks .
baby and complete delivery of placenta , started B . Gestational age 14 _ 16 weeks .
C . Gestational age 20 _ 24 weeks .
‫ اﺳﺎﻣﻪ اﻟﺨﺰاﻋﻠﻪ‬: ‫اﻟﺪﻛﺘﻮر‬ ‫اﺳﺌﻠﺔ اﻟﻨﺴﺎﺋﻴﺔ واﻟﺘﻮﻟﻴﺪ‬
Page 41
D . Gestational age 24 _ 28 weeks .
E . Gestational age 18 _ 22 weeks .

* All of the following are relative


contraindications for combined oral
contraceptive pills except ?
A . Gallbladder disease .
B . DM with vascular disease .
C . Major surgeries with prolonged
immobilization
D . Migraine and age over 35 years of age .
E . Family history of DVT in first degree relative
under the age of 45 years .

‫ اﺳﺎﻣﻪ اﻟﺨﺰاﻋﻠﻪ‬: ‫اﻟﺪﻛﺘﻮر‬ ‫اﺳﺌﻠﺔ اﻟﻨﺴﺎﺋﻴﺔ واﻟﺘﻮﻟﻴﺪ‬


Page 42

You might also like